Вы находитесь на странице: 1из 97

www.insightsonindia.

com www.insightsias.com
TEST – 13 Solutions

Prelims 2017 – Test 13


SOLUTIONS

1. Consider the following statements about Aranyakas.


1. They are the concluding portions of the several Brahmanas.
2. They mainly deal with sacrificial techniques and karma kandas.
3. These were works to be read in the villages, as opposed to
‘Brahmanas’ text which must be read in the forests.
4. There is no Aranyaka which belongs to the Atharvaveda.

Select the correct answer using the codes below.

a) 1 and 4 only
b) 1 and 2 only
c) 3 only
d) 1, 3 and 4 only

Solution: a)

Justification: Statement 1: Aranyakas are generally the concluding


portions of the several Brahmanas, but on account of their distinct
character, contents and language deserve to be reckoned as a distinct
category of literature. They are partly included in the Brahmanas
themselves, but partly they are recognized as independent works.

Statement 2: Aranyaka literature is rather small as compared to the


Brahmanas. Whereas the Brahmanas deal with the huge bulk of
sacrificial paraphernalia which represents Karma-Kanda, the Aranyakas
and Upanishads, on the other hand, chiefly deal with the philosophical
and theosophical speculations which represent Jnana-Kanda.

Statement 3: The term Aranyaka is derived from the word 'Aranya'


meaning 'forest'. The Aranyaka texts are so-called because 'they were


INSIGHTS PRELIMS TEST SERIES 2017

www.insightsonindia.com www.insightsias.com
TEST – 13 Solutions

works to be read in the forest' in contradistinction to the regular


Brahmanas, which were to be read in the village.

• This is because Yajna and other rituals are prescribed only for
those who live in homes and lead the life of house-holders. But it
has to be understood that Vedic rituals are intended to confer not
only material benefits but also mental purity by constant
discipline. Having obtained purity, one must seek the solitude of
forests for further concentration and meditation.

• Aranyakas containing explanations of the rituals and allegorical


speculations meant for Vanprasthas, who renounce family life
residing in the forests for tapas and other religious activities.

Statement 4: Aranayakas have been written for Rigvedal; Samaveda;


Shukla Yajurveda; and Krishna Yajurveda only.

Q Source: Page 28: 11th Standard TamilNadu History Textbook

2. Consider the following statements.

Assertion (A): Real interest rates have turned negative many times in
India.

Reason (R): High inflation has been registered in many financial years.

In the context of the above, which of these is correct?

a) A is correct, and R is an appropriate explanation of A.


b) A is correct, but R is not an appropriate explanation of A.
c) A is incorrect, but R is correct.
d) Both A and R are incorrect.

Solution: a)

Justification: Nominal interest rates are what banks charge you on


lending. But, this is not the actual interest rates received by banks,
because inflation erodes earnings on interests.


INSIGHTS PRELIMS TEST SERIES 2017

www.insightsonindia.com www.insightsias.com
TEST – 13 Solutions

Real interest rate is the nominal rate adjusted for inflation. Suppose if
nominal rate is 10% and inflation is 8%, real interest rates will be 2%
only. But, if inflation exceeds 10% mark, real interest rate will turn
negative.

India has witnessed negative real interest rates since 2008, which has
disincentivized savers, thereby triggering a sharp movement of
household savings (70% of overall savings) from financial assets to
physical assets. This is due to the double digit WPI and CPI inflation
witnessed in these years. So, A is incorrect, but R is correct.

Q Source: Page 39: Chapter 3: 12th Macroeconomics NCERT

3. The Parliament can redraw the political map of India according to


its will. Which of these arguments or statements support this view?
1. Parliament is not bound by the views of the state legislature and
may either accept or reject them.
2. The constitution must be amended under Article 368 to
accommodate new states, for which states do not play any
decisive role.

Which of the above is/are correct?

a) 1 only
b) 2 only
c) Both 1 and 2
d) None

Solution: a)

Justification: Statement 1: Article 3 authorises the Parliament to form


a new state or change area of an existing state.

However, Article 3 lays down two conditions in this regard: one, a bill
contemplating the above changes can be introduced in the Parliament


INSIGHTS PRELIMS TEST SERIES 2017

www.insightsonindia.com www.insightsias.com
TEST – 13 Solutions

only with the prior recommendation of the President; and two, before
recommending the bill, the President has to refer the same to the state
legislature concerned for expressing its views within a specified period.

But, the President (or Parliament) is not bound by the views of the state
legislature and may either accept or reject them, even if the views are
received in time.

Statement 2: Creation of new states does not require constitutional


amendment as such amendments are not to be deemed as
“amendments” under A368. So, 2 is wrong.

Q Source: Chapter 5: Indian Polity: M Laxmikanth

4. Consider the following statements.


1. The writ of Prohibition can be issued only against judicial and
administrative authorities.
2. The writ of Certiorari is different from Prohibition in that the
former can be applied even to legislative bodies that exceed
their jurisdiction.

Which of the above is/are correct?

a) 1 only
b) 2 only
c) Both 1 and 2
d) None

Solution: d)

Justification: Statement 1: Prohibition writ is issued by a higher court


to a lower court or tribunal to prevent the latter from exceeding its


INSIGHTS PRELIMS TEST SERIES 2017

www.insightsonindia.com www.insightsias.com
TEST – 13 Solutions

jurisdiction or usurping a jurisdiction that it does not possess. Thus,


unlike mandamus that directs activity, the prohibition directs inactivity.

The writ of prohibition can be issued only against judicial and quasi-
judicial authorities. It is not available against administrative authorities,
legislative bodies, and private individuals or bodies.

Statement 2: Certiorari in the literal sense, means ‘to be certified’ or ‘to


be informed’. It is issued by a higher court to a lower court or tribunal
either to transfer a case pending with the latter to itself or to squash the
order of the latter in a case. It is issued on the grounds of excess of
jurisdiction or lack of jurisdiction or error of law.

Like prohibition, certiorari is also not available against legislative bodies


and private individuals or bodies. So, 2 is wrong.

Thus, unlike prohibition, which is only preventive, certiorari is both


preventive as well as curative.

Learning: Till recently, the writ of certiorari could be issued only


against judicial and quasi-judicial authorities and not against
administrative authorities. However, in 1991, the Supreme Court ruled
that the certiorari can be issued even against administrative authorities
affecting rights of individuals.

Q Source: Chapter 7: Indian Polity: M Laxmikanth

5. It is argued that GDP does not cover up externalities in production


of goods and services. If there are positive externalities then
a) The socially desirable GDP will be lower than commercially
desirable GDP
b) The socially desirable GDP will be higher than commercially
desirable GDP
c) The socially desirable GDP cannot be produced with the
intervention of the state.


INSIGHTS PRELIMS TEST SERIES 2017

www.insightsonindia.com www.insightsias.com
TEST – 13 Solutions

d) The commercially desirable GDP will not be able to utilize all


economic resources efficiently.

Solution: b)

Learning: Consumption, production, and investment decisions of


individuals, households, and firms often affect people not directly
involved in the transactions. Sometimes these indirect effects are tiny.

• But when they are large they can become problematic—what


economists call externalities. Externalities are among the main
reasons governments intervene in the economic sphere.

• In the case of pollution—the traditional example of a negative


externality—a polluter makes decisions based only on the direct
cost of and profit opportunity from production and does not
consider the indirect costs to those harmed by the pollution. The
indirect costs include decreased quality of life, say in the case of a
home owner near a smokestack; higher health care costs; and
forgone production opportunities, for example, when pollution
harms activities such as tourism.

• Since the indirect costs are not borne by the producer, and
therefore not passed on to the end user of the goods produced by
the polluter, the social or total costs of production are larger than
the private costs.

• There are also positive externalities, and here the issue is the
difference between private and social gains. For example, research
and development (R&D) activities are widely considered to have
positive effects beyond those enjoyed by the producer that funded
the R&D—normally, the company that pays for the research.

• This is because R&D adds to the general body of knowledge, which


contributes to other discoveries and developments. However, the
private returns of a firm selling products based on its own R&D
typically do not include the returns of others who benefited
indirectly. With positive externalities, private returns are smaller
than social returns.

Q Source: Page 30: 12th Macroeconomics NCERT


INSIGHTS PRELIMS TEST SERIES 2017

www.insightsonindia.com www.insightsias.com
TEST – 13 Solutions

6. Which of these are constitutional provisions?


1. Duties of prime minister to furnish required information to the
president
2. Power of president to consult Supreme Court
3. Special provisions with respect to Delhi

Select the correct answer using the codes below.

a) 1 and 2 only
b) 2 only
c) 1 and 3 only
d) 1, 2 and 3

Solution: d)
Justification: Statement 1: It is dealt by A78.

Duties of Prime Minister as respects the furnishing of information to the


President, etc It shall be the duty of the Prime Minister

a) to communicate to the President all decisions of the council of


Ministers relating to the administration of the affairs of the union
and proposals for legislation;
b) to furnish such information relating to the administration of the
affairs of the Union and proposals for legislation as the President
may call for; and
c) if the President so requires, to submit for the consideration of the
Council of Ministers any matter on which a decision has been
taken by a Minister but which has not been considered by the
Council

Statement 2: A143: Power of President to consult Supreme Court: If at


any time it appears to the President that a question of law or fact has
arisen, or is likely to arise, which is of such a nature and of such public
importance that it is expedient to obtain the opinion of the Supreme
Court upon it, he may refer the question to that Court for consideration


INSIGHTS PRELIMS TEST SERIES 2017

www.insightsonindia.com www.insightsias.com
TEST – 13 Solutions

and the Court may, after such hearing as it thinks fit, report to the
President its opinion thereon

Statement 3: Under article 239 AA the legislative assembly of National


Capital Territory of Delhi has legislative and executive powers with the
exceptions of land, police etc. v

Also, certain categories of Bills require the prior approval of the Central
Government for introduction in the legislative assembly. Some Bills,
passed by the legislative assembly of the Union Territory of Puducherry
and National Capital Territory of Delhi are required to be reserved for
consideration and assent of the President.

Q Source: Chapter 3: Indian Polity: M Laxmikanth

7. How would you distinguish between the revenue and capital


receipts of the government?
1. Revenue receipts are non-redeemable unlike certain capital
receipts.
2. Capital receipts are always debt creating unlike revenue
receipts.

Which of the above is/are correct?

a) 1 only
b) 2 only
c) Both 1 and 2
d) None

Solution: a)
Justification: Statement 1: The main difference between revenue
receipts and capital receipts is that in the case of revenue receipts,
government is under no future obligation to return the amount, i.e., they
are non-redeemable. But In case of capital receipts which are


INSIGHTS PRELIMS TEST SERIES 2017

www.insightsonindia.com www.insightsias.com
TEST – 13 Solutions

borrowings, government is under obligation to return the amount along


with Interest.

Statement 2: Capital receipts may be debt creating or non-debt creating.


So, 2 is wrong.

Examples of debt creating receipts are—Net borrowing by government at


home, loans received from foreign governments, borrowing from RBI.
Examples of non-debt capital receipts are—Recovery of loans, proceeds
from sale of public enterprises (i.e., disinvestment), etc. These do not
give rise to debt.

Q Source: Improvisation: Chapter 5: 12th Macroeconomics NCERT

8. The landmark judgment in ‘Coelho case’ dealt with


a) Inclusion of Ninth Schedule under the ambit of Judicial Review
b) Limitations of the power of pardon to President and Governors
c) Power of Parliament to amend the Preamble
d) Conflict between certain minority religious practices and their
fundamental rights

Solution: a)
Learning: The first amendment to the Indian Constitution added the
Ninth Schedule to it. It was introduced to address judicial review
decisions especially about abridging certain fundamental rights.

The state wanted to pursue nationalisation, take away lands from the
zamindars, re-distribute them, and make special provisions for the
socially and economically backward. This would have violated certain
FRs, thus the need for 9th schedule to shield executive and legislative
actions from Judicial review.

I.R. Coelho by L.Rs. v. State of Tamil Nadu case raised the following
questions: Is it permissible to make the 9th Schedule immunized from


INSIGHTS PRELIMS TEST SERIES 2017

www.insightsonindia.com www.insightsias.com
TEST – 13 Solutions

the Judicial Review of the Supreme Court? Whether the Basic Structure
test would include Judicial Review of Ninth Schedule laws on the
touchstone of Fundamental rights?

The court ruled that it is absolutely not permissible to make the Ninth
Schedule immunized from the Judicial Review of the Constitution.

The court said that the Doctrine of Basic Structure is the very essence of
the Constitution of India and therefore, there cannot be any act, rules or
regulations which can overrule the Basic Structure doctrine.

Q Source: Improvisation: Chapter 7: Indian Polity: M Laxmikanth

9. Consider the following statements.


1. The Indian Councils Act 1861 transformed the executive council
of Viceroy into a cabinet run on the portfolio system.
2. The members of the interim government (1946) were members
of the Viceroy’s Executive Council.

Which of the above is/are correct?

a) 1 only
b) 2 only
c) Both 1 and 2
d) None

Solution: c)
Justification: Statement 1: The Viceroy's Executive Council was the
cabinet of the government of British India headed by the Viceroy of
India. It was transformed from an advisory council into a cabinet run by
the portfolio system by the Indian Councils Act 1861.

Statement 2: The interim government of India was formed in 1946 from


the newly elected Constituent Assembly.


INSIGHTS PRELIMS TEST SERIES 2017

www.insightsonindia.com www.insightsias.com
TEST – 13 Solutions

The members of the interim government also include the Viceroy and
Commander-in-Chief of Britain and other members from All-India
Muslim League. The members of the interim government were also the
members of the Viceroy’s Executive Council.

Q Source: Improvisation: Chapter 2: Indian Polity: M Laxmikanth

10. Consider the following about “Habitat III” conference.


1. It is also known as UN Conference on Housing and Sustainable
Urban Development.
2. The 2016 conference was held at New Delhi.
3. The “New Urban Agenda” adopted at the conference commits
development financing by developed countries for urbanization
of least developed countries (LDCs).

Select the correct answer using the codes below.

a) 1 and 2 only
b) 2 and 3 only
c) 1 only
d) 1, 2 and 3

Solution: c)
Justification: The New Urban Agenda was adopted at the UN
Conference on Housing and Sustainable Urban Development (also
known as “Habitat III” conference) held in Quito, Ecuador. So, 1 is
correct and 2 is wrong.

It sets the global vision of sustainable urbanization for the next 20 years.
It is a set of 175 commitments that countries need to adhere to tackle
challenges of urbanization.


INSIGHTS PRELIMS TEST SERIES 2017

www.insightsonindia.com www.insightsias.com
TEST – 13 Solutions

It is considered as roadmap for building cities that can serve as engines


of prosperity and centres of social and cultural well-being while
protecting the environment. It provides guidance for achieving the
Sustainable Development Goals and provides the underpinning for
actions to address climate change.

So, 3 is wrong as there is no commitment by developed countries to fund


urbanization in LDCs.

Q Source: Major environmental conferences of UN

11. The ‘August Offer’ of 1940 accepted which of these important


demands of the Indian National Congress leadership?
a) Rejection of the idea of partition of India
b) Establishing a system of provincial elections and giving them
greater autonomy
c) Constituent Assembly of India elected on the basis of adult
franchise
d) Withdrawal of involvement of Indian troops in the Second
World War

Solution: c)

Learning: It was in 1934 that the idea of a Constituent Assembly for


India was put forward for the first time by M. N. Roy, a pioneer of
communist movement in India and an advocate of radical democratism.

In 1935, the Indian National Congress (INC), for the first time, officially
demanded a Constituent Assembly to frame the Constitution of India.

In 1938, Jawaharlal Nehru, on behalf the INC declared that ‘the


Constitution of free India must be framed, without outside interference,
by a Constituent Assembly elected on the basis of adult franchise’.


INSIGHTS PRELIMS TEST SERIES 2017

www.insightsonindia.com www.insightsias.com
TEST – 13 Solutions

The demand was finally accepted in principle by the British Government


in what is known as the ‘August Offer’ of 1940.

Q Source: Chapter 2: Indian Polity: M Laxmikanth

12.The Constituent Assembly was constituted in November 1946


under the scheme formulated by the Cabinet Mission Plan. What
was/were the salient features of the assembly?
1. The assembly had representation from both British India and
princely states.
2. The system of proportional representation was adopted.
3. There was no communal representation in the assembly.
4. The Assembly included important ministers of the British
Cabinet as ex-officio members.

Select the correct answer using the codes below.

a) 1 and 2 only
b) 1 and 3 only
c) 2 and 4 only
d) 1, 2 and 4 only

Solution: a)

Justification: Statement 1: The total strength of the Constituent


Assembly was to be 389. Of these, 296 seats were to be allotted to British
India and 93 seats to the Princely States. Out of 296 seats allotted to the
British India, 292 members were to be drawn from the eleven governors’
provinces2 and four from the four chief commissioners’ provinces3, one
from each.

Statement 2: Each province and princely state (or group of states in case
of small states) were to be allotted seats in proportion to their respective
population. Roughly, one seat was to be allotted for every million
population.


INSIGHTS PRELIMS TEST SERIES 2017

www.insightsonindia.com www.insightsias.com
TEST – 13 Solutions

Statement 3: Seats allocated to each British province were to be decided


among the three principal communities—Muslims, Sikhs and general (all
except Muslims and Sikhs), in proportion to their population.

The representatives of each community were to be elected by members


of that community in the provincial legislative assembly and voting was
to be by the method of proportional representation by means of single
transferable vote.

Statement 4: There was no such representation.

It included all important personalities of India at that time, with the


exception of Mahatma Gandhi and M A Jinnah.

Learning: The representatives of princely states were to be nominated


by the heads of the princely states. It is thus clear that the Constituent
Assembly was to be a partly elected and partly nominated body.

Moreover, the members were to be indirectly elected by the members of


the provincial assemblies, who themselves were elected on a limited
franchise4.

Q Source: Chapter 2: Indian Polity: M Laxmikanth

13. Consider the following about the early Rig Vedic society.
1. It was patriarchal in nature.
2. Child marriage was highly prevalent.
3. The practice of sati was absent.
4. Polygamy was prohibited due to religious reasons.

Select the correct answer using the codes below.

a) 1 and 3 only
b) 2 and 4 only
c) 1, 3 and 4 only
d) 1, 2 and 3 only


INSIGHTS PRELIMS TEST SERIES 2017

www.insightsonindia.com www.insightsias.com
TEST – 13 Solutions

Solution: a)

Justification: There was no child marriage and the practice of sati was
absent. However, the society was patriarchal. So, 1 and 3 are correct and
2 is incorrect.

The basic unit of society was family or graham. The head of the family
was known as grahapathi. Monogamy was generally practiced while
polygamy was prevalent among the royal and noble families. So, 4 is
incorrect.

The wife took care of the household and participated in all the major
ceremonies. Women could even attend the popular assemblies.

Also, there were women poets like Apala, Viswavara, Ghosa and
Lopamudra during the Rig Vedic period.

Q Source: Page 29: 11th Standard TamilNadu History Textbook

14.The Cabinet Mission included which of these as members?


1. Mohandas Karamchand Gandhi
2. Mountbatten
3. Stafford Cripps

Select the correct answer using the codes below.

a) 2 only
b) 1 and 3 only
c) 3 only
d) None of the above

Solution: c)

Justification: The Cabinet Mission was sent to India to negotiate the


constitutional position of India and the future of British empire in India.


INSIGHTS PRELIMS TEST SERIES 2017

www.insightsonindia.com www.insightsias.com
TEST – 13 Solutions

It consisted of three members - Lord Pethick Lawrence, Sir Stafford


Cripps and A V Alexander. The Cabinet Mission published its plan on
1946.

It was notable that there was no Indian member in the Mission.

We will be covering more questions on this as it is an important topic.

Q Source: Chapter 2: Indian Polity: M Laxmikanth

15. Consider the following ‘Conflict prone regions in news’ with the
nation they are located in.
1. Aleppo: Iraq
2. Boko Haram: Nigeria
3. Ramadi: Syria

Which of the above is/are correctly matched?

a) 1 only
b) 2 only
c) 1 and 3 only
d) 2 and 3 only

Solution: b)

Justification: Statement 1: Aleppo was Syria’s largest city and


commercial capital before the conflict broke out. The Battle of Aleppo
lasted more than four years after fighting broke out in July 2012.
Recently Syrian Army has officially liberated Aleppo after ousting rebels
from their last bastions in the eastern part of the city.

Statement 2: The Boko Haram insurgency began in 2009, when the


jihadist rebel group Boko Haram started an armed rebellion against the
government of Nigeria.


INSIGHTS PRELIMS TEST SERIES 2017

www.insightsonindia.com www.insightsias.com
TEST – 13 Solutions

Statement 3: ISIL forces have seized large parts of Iraq - Anbar


province, including the cities of Fallujah, Al Qaim, Abu Ghraib and half
of Ramadi during the Anbar campaign.

Tikrit, Mosul and other parts too were seized by insurgent forces.

Q Source: Frequently in news

16.The Union AYUSH Ministry has recently launched the ‘Swasthya


Raksha Programme’ to promote
a) Indian Medicine system on a mission mode in urban areas
b) Early tracking of development defects in children
c) Health and health education in villages
d) Training of MBBS practitioners for serving in rural areas

Solution: c)

Learning: The programme was initiated through Central Council for


Research in Ayurvedic Sciences (CCRAS) and other bodies.

• It was launched by AYUSH Ministry in October 2015.


• The programme will organize Swasthya Parikshan Camps,
Swasthya Rakshan OPDs and Health and Hygiene awareness
programme Create awareness about cleanliness of domestic
surroundings and environment.
• It will provide medical aid and incidental support in the adopted
villages and colonies.
• It will document demographic information, hygiene conditions,
food habits, seasons, lifestyle etc., incidence and prevalence of
disease and their relation to the incidence of disease.
• It will further assess health status and propagation of Ayurvedic
concept of pathya-apathya and extension of health care services.

Q Source: Major schemes: PIB


INSIGHTS PRELIMS TEST SERIES 2017

www.insightsonindia.com www.insightsias.com
TEST – 13 Solutions

17. The historic ‘Objectives Resolution’ was passed in


a) Lahore Session of the Indian National Congress
b) Constituent Assembly in 1946
c) All Parties Conference called after the Cabinet Mission Plan
d) A Meeting which declared the Mountbatten Plan

Solution: b)

Learning: In December, 1946, Jawaharlal Nehru moved the historic


‘Objectives Resolution’ in the Assembly. It laid down the fundamentals
and philosophy of the constitutional structure. It read:

It included the major values and ideals of sovereignty, republic,


fundamental rights, directive principles, non-interference etc.

It sought to secure to ideals mentioned in the Preamble.

It provided for adequate safeguards for minorities, backward and tribal


areas, and depressed and other backward classes.

This Resolution was unanimously adopted by the Assembly on January


22, 1947. It influenced the eventual shaping of the constitution through
all its subsequent stages. Its modified version forms the Preamble of the
present Constitution.

Q Source: Chapter 2: Indian Polity: M Laxmikanth


18. The Fundamental Rights promote the idea of political


democracy as
1. They limit the authority of the Central government.
2. They secure vital political rights to the citizens of India.

Which of the above is/are correct?


INSIGHTS PRELIMS TEST SERIES 2017

www.insightsonindia.com www.insightsias.com
TEST – 13 Solutions

a) 1 only
b) 2 only
c) Both 1 and 2
d) None

Solution: c)

Justification: They operate as limitations on the tyranny of the


executive and arbitrary laws of the legislature. They are justiciable in
nature, that is, they are enforceable by the courts for their violation.

Statement 2: Rights such as equality to contest for political office, right


against discrimination etc show the political and social equality of
citizens.

Learning: The aggrieved person can directly go to the Supreme Court


which can issue the writs of habeas corpus, mandamus, prohibition,
certiorari and quo warranto for the restoration of his rights.

However, the Fundamental Rights are not absolute and subject to


reasonable restrictions. Further, they are not sacrosanct and can be
curtailed or repealed by the Parliament through a constitutional
amendment act. They can also be suspended during the operation of a
National Emergency except the rights guaranteed by Articles 20 and 21.

Q Source: Chapter 3: Indian Polity: M Laxmikanth

19.Lothal was a vital and thriving trade centre in ancient times. It


formed a trade link between
1. Harappan cities in Sindh and the peninsula of Saurashtra
2. Mesopotamia and Indus valley civilization

Which of the above is/are correct?

a) 1 only
b) 2 only
c) Both 1 and 2
d) None

Solution: c)


INSIGHTS PRELIMS TEST SERIES 2017

www.insightsonindia.com www.insightsias.com
TEST – 13 Solutions

Justification: Statement 1: Lothal was a major trade centre, importing


huge raw materials like copper, chert and semi-precious stones from
Mohenjo-daro and Harappa, and mass distributing to inner villages and
towns.

Statement 2: An intensive trade network gave the inhabitants great


prosperity. The network stretched across the frontiers to Egypt, Bahrain
and Sumer. One of the evidence of trade in Lothal is the discovery of
typical Persian gulf seals, a circular button seal.

Lothal remained an emporium of trade between the Harappan


civilization and the remaining part of India as well as Mesopotamia.

Q Source: Page 18: 11th Standard TamilNadu History Textbook

20. Which of these enactments by the British Parliament


provided the British Government ultimate control over Company’s
affairs and its administration in India?
a) Indian Councils Act 1861
b) Charter Act of 1853
c) Pitt’s India Act of 1784
d) All of the above are incorrect as they were legislated by the
Central Legislative Assembly and not the British Parliament.

Solution: c)

Learning: It was the Pitt’s India Act of 1784, but it was passed by the
British Parliament, not the CLA of colonial India. Hence, D is incorrect,
and A will be the correct option.

In a bid to rectify the defects of the Regulating Act of 1773, the British
Parliament passed the Amending Act of 1781, also known as the Act of
Settlement. The next important act was the Pitt’s India Act.

1. It distinguished between the commercial and political functions of


the Company.


INSIGHTS PRELIMS TEST SERIES 2017

www.insightsonindia.com www.insightsias.com
TEST – 13 Solutions

2. It allowed the Court of Directors to manage the commercial affairs


but created a new body called Board of Control to manage the
political affairs. Thus, it established a system of double
government.
3. It empowered the Board of Control to supervise and direct all
operations of the civil and military government or revenues of the
British possessions in India.

Thus, the act was significant for two reasons: first, the Company’s
territories in India were for the first time called the ‘British possessions
in India’; and second, the British Government was given the supreme
control over Company’s affairs and its administration in India.

Q Source: Chapter 1: Indian Polity: M Laxmikanth

21.Suppose India manufactures only Cars and United States


manufactures only Clothes. Both nations float their currency. In
such a scenario, the exchange rate between Indian and US
currency will depend on
1. Productivity of labour and capital in India and USA
2. Demand for Indian cars in USA
3. Demand for USA manufactured clothes in India

Select the correct answer using the codes below.

a) 1 and 2 only
b) 2 and 3 only
c) 1 only
d) 1, 2 and 3

Solution: d)

Justification: Exchange rate of a currency basically depends on the


supply and demand of the currency.


INSIGHTS PRELIMS TEST SERIES 2017

www.insightsonindia.com www.insightsias.com
TEST – 13 Solutions

These two in turn are affected by export/import demand, political


stability of a nation, its income etc.

Statement 1: Suppose if labour and capital are highly productive in India,


this will increase the overall productivity of firms. This will lead to low
cost products as well as high wages for labour and high returns for
capital owners. Increased income will push up import demand and thus
affect exchange rates.

Same argument can be given for the opposite case.

Statement 2: Higher demand for Indian cars will mean India’s exports
are large, which in turn implies a high demand for Indian currency by
foreigners (US Residents) and thus a highly valued rupee as compared to
the Dollar.

Statement 3: Same argument applies here too.

Q Source: Page 79: 12th Macroeconomics NCERT

22. As you flip through the pages of the constitution, you are
likely to find specific provisions for the following in which order?
1. Legislative Powers of the President
2. Comptroller and Auditor General of India
3. High courts
4. Citizenship

Select the correct answer using the codes below.

a) 4312
b) 1234
c) 4123
d) 1324

Solution: c)


INSIGHTS PRELIMS TEST SERIES 2017

www.insightsonindia.com www.insightsias.com
TEST – 13 Solutions

Justification: If you remember some major articles of the Constitution,


you can arrive at the answer. Or if you know broadly which parts of the
Constitution deal with which subjects, you can arrive at the correct
answer.

The following shows the right order.

Q Source: Chapter 3: Indian Polity: M Laxmikanth


INSIGHTS PRELIMS TEST SERIES 2017

www.insightsonindia.com www.insightsias.com
TEST – 13 Solutions

23. Which of these acts, for the first time, provided for the
association of Indians with the executive Councils of the Viceroy
and introduced the system of communal representation?
a) Act of 1909
b) Act of 1919
c) Government of India Act 1935
d) Councils Act 1891

Solution: a)

Learning: This Act is also known as Morley-Minto Reforms (Lord


Morley was the then Secretary of State for India and Lord Minto was the
then Viceroy of India).

The Indian Councils Act 1909 empowered the Governor General to


nominate one Indian member to the Executive Council leading to the
appointment of Satyendra Prasanno Sinha as the first Indian member.
The Government of India Act 1919 increased the number of Indians in
the council to three.

It introduced a system of communal representation for Muslims by


accepting the concept of ‘separate electorate’. Under this, the Muslim
members were to be elected only by Muslim voters. Thus, the Act
‘legalised communalism’ and Lord Minto came to be known as the
Father of Communal Electorate.

It also provided for the separate representation of presidency


corporations, chambers of commerce, universities and zamindars.

Q Source: Chapter 1: Indian Polity: M Laxmikanth

24. Gender Budgeting is a means of ensuring more equitable


allocation of public resources to specific gender groups. As per the
2007 guidelines issued by the Ministry of Finance, a Gender
Budgeting Cell (GBC) should consist of
1. At least one women rights activist
2. One or more women Member of Parliament (MP)
3. An officer of the level of Joint Secretary


INSIGHTS PRELIMS TEST SERIES 2017

www.insightsonindia.com www.insightsias.com
TEST – 13 Solutions

Select the correct answer using the codes below.

a) 1 and 2 only
b) 3 only
c) 2 and 3 only
d) 2 only

Solution: b)

Justification: With the objective of facilitating the integration of


gender analysis into the Government budget, a charter for the Gender
Budgeting Cells was published for guidance and implementation by all
Ministries/ Departments.

As per the guidelines “The Gender Budget Cell should comprise a


cohesive group of senior/ middle level officers from the Plan, Policy,
Coordination, Budget and Accounts Division of the Ministry concerned.

This group should be headed by an officer not below the rank of Joint
Secretary. The functions and working of the GRB may be reviewed at
least once a quarter at the level of Secretary/ Additional Secretary of the
Department.”

Q Source: Improvisation: Page 64: 12th Macroeconomics NCERT

25. Which of the following words are attached to the term


“Republic” in the Premble?
1. Sovereign
2. Parliamentary
3. Secular
4. Federal

Select the correct answer using the codes below.

a) 1 and 2 only
b) 1 and 3 only
c) 2, 3 and 4 only


INSIGHTS PRELIMS TEST SERIES 2017

www.insightsonindia.com www.insightsias.com
TEST – 13 Solutions

d) 1, 2, 3 and 4

Solution: b)

Justification: “We, THE PEOPLE OF INDIA, having solemnly resolved


to constitute India into a SOVEREIGN SOCIALIST SECULAR
DEMOCRATIC REPUBLIC and to secure to all its citizens:

JUSTICE, Social, Economic and Political;

LIBERTY of thought, expression, belief, faith and worship;

EQUALITY of status and of opportunity; and to promote among them


all;

FRATERNITY assuring the dignity of the individual and the unity and
integrity of the Nation;

IN OUR CONSTITUENT ASSEMBLY this twenty-sixth day of November,


1949, do HEREBY ADOPT, ENACT AND GIVE TO OURSELVES THIS
CONSTITUTION”

Q Source: Chapter 4: Indian Polity: M Laxmikanth

26. Avadi session of Indian National Congress was famously


known for
1. Adopting a resolution to establish a ‘socialistic pattern of
society’
2. Publishing the ‘Magna carta’ of a libertarian democratic
framework for India

Which of the above is/are correct?

a) 1 only
b) 2 only
c) Both 1 and 2
d) None


INSIGHTS PRELIMS TEST SERIES 2017

www.insightsonindia.com www.insightsias.com
TEST – 13 Solutions

Solution: a)

Justification: The famous session of the Indian National Congress was


held at Avadi in January 1955.

This historical meet emphasised the importance of socialism and its


impact on social development.

Jawaharlal Nehru with Morarji Desai and other Congress leaders at the
AICC session declared that a socialistic pattern of society was the goal of
the Congress.

This was the basic philosophy of the INC- “It is not possible to pursue a
policy of laissez-faire in industry.... It is incompatible with any planning.
It has long been Congress policy that basic industries should be owned or
controlled by the state.”

“This policy holds and must be progressively given effect to. State trading
should be undertaken wherever the balance of advantage lies in favour of
such a course. A large field for private enterprise is, however, left over.
Thus our economy will have a public sector as well as a private sector.
But the private sector must accept the objective of the National Plan and
fit into it.”

Q Source: Chapter 4: Indian Polity: M Laxmikanth

27. Sterilization of foreign inflows by RBI is done with a view to


1. Control money supply
2. Control inflow of financial services in India

Which of the above is/are correct?

a) 1 only
b) 2 only
c) Both 1 and 2
d) None


INSIGHTS PRELIMS TEST SERIES 2017

www.insightsonindia.com www.insightsias.com
TEST – 13 Solutions

Solution: a)

Justification: Entry or outgo of foreign exchange in India has the


potential to alter total money supply.

This is because the inflow must be converted into Indian currency, and
the outflow must be converted from Indian currency back to foreign
exchange.

RBI undertakes an open market sale of government securities of an


amount equal to the amount of foreign exchange inflow in the economy,
thereby keeping the stock of high powered money and total money
supply unchanged.

Thus it sterilises the economy against adverse external shocks. This


operation of RBI is known as sterilisation.

Statement 2: RBI does not interfere with the pattern of trade as such,
and thus statement 2 will be wrong.

Q Source: Page 47: 12th Macroeconomics NCERT

28. India ceased to be a British dominion on


a) The passage of the Indian Independence Act, 1947
b) August 15, 1947
c) January 26, 1950
d) Day of swearing in of Independent India’s first Governor-
General

Solution: c)

Justification & Learning: Till the passage of the Indian


Independence Act, 1947, India was a dependency (colony) of the British
Empire. From August 15, 1947 to January 26, 1950, India’s political
status was that of a dominion in the British Commonwealth of Nations.


INSIGHTS PRELIMS TEST SERIES 2017

www.insightsonindia.com www.insightsias.com
TEST – 13 Solutions

India ceased to be a British dominion on January 26, 1950, by declaring


herself a sovereign republic. However, Pakistan continued to be a British
Dominion until 1956.

India also decided to become a member of the commonwealth, however


this did not affect India’s independence.

Q Source: Chapter 4: Indian Polity: M Laxmikanth

29. If the Monetary Policy Committee (MPC) increases the Cash


Reserve Ratio (CRR), it may lead to
a) Lower interest rates
b) Reduction in fiscal deficit
c) Immediate foreign portfolio flows
d) Lower lending by banks

Solution: d)
Justification: If RBI decides to increase CRR, banks will keep more
cash with RBI and lend less. Lower money circulation would reduce
economic activity. So, D is correct.

Fiscal deficit depends on government’s receipts and expenditures. CRR


does not have a direct bearing on fiscal deficit. So, B is wrong.

FPI depends on interest rates in the country and general investment


climate. Even though high CRR may push up interest rates, the
transmission is very slow as interest rates do not change suddenly due to
a CRR hike. So, C is inappropriate.

Q Source: Page 41: 12th Macroeconomics NCERT


INSIGHTS PRELIMS TEST SERIES 2017

www.insightsonindia.com www.insightsias.com
TEST – 13 Solutions

30. Which of these provisions of the Constitution reveal the


secular character of the Indian State?
1. Preamble
2. Directive Principles of State Policy
3. Fundamental Rights

Select the correct answer using the codes below.

a) 2 only
b) 1 and 3 only
c) 2 and 3 only
d) 1, 2 and 3

Solution: d)
Justification: Statement 1: The term ‘secular’ was added to the
Preamble of the Indian Constitution by the 42nd Constitutional
Amendment Act of 1976. The Preamble secures to all citizens of India
liberty of belief, faith and worship.

Statement 2: The State shall endeavour to secure for all the citizens a
Uniform Civil Code (Article 44) is mentioned in DPSP – Part IV.

Statement 3: The State shall not deny to any person equality before the
law or equal protection of the laws (Article 14).

• The State shall not discriminate against any citizen on the ground
of religion (Article 15).
• Equality of opportunity for all citizens in matters of public
employment (Article 16).
• All persons are equally entitled to freedom of conscience and the
right to freely profess, practice and propagate any religion (Article
25).
• Every religious denomination or any of its section shall have the
right to manage its religious affairs (Article 26).
• Similarly Articles 27-30 also uphold values of secularism.

Q Source: Chapter 3: Indian Polity: M Laxmikanth


INSIGHTS PRELIMS TEST SERIES 2017

www.insightsonindia.com www.insightsias.com
TEST – 13 Solutions

31. The metaphorical "invisible hand" in Economics is used to refer to


1. Forces of free market
2. Socialist orientation of state
3. Public private partnership

Select the correct answer using the codes below.

a) 1 only
b) 2 and 3 only
c) 1 and 3 only
d) 2 only

Solution: a)
Justification: Classical economists observe that markets generally
regulate themselves, when free of coercion. Adam Smith referred to this
as a metaphorical "invisible hand," which moves markets toward their
natural equilibrium.

So, classical thought is also known as the non-interventionist school. As


per them, markets function best without government interference.

In contrast to classical economics, Keynesian economics supports


policies such as government intervention, deficit spending, control of the
money supply, and a progressive income tax to counter recession and
income inequality.

Q Source: Page 5: 12th Macroeconomics NCERT

32. Which of these metals were NOT known to the Rig Vedic
people?
a) Gold
b) Iron
c) Silver
d) Aluminium

Solution: d)


INSIGHTS PRELIMS TEST SERIES 2017

www.insightsonindia.com www.insightsias.com
TEST – 13 Solutions

Learning: Apart from copper, bronze, and gold, later Vedic texts also
mention tin, lead, and silver.

Metallurgy is not mentioned in the Rig Veda, but the word ayas and
instruments made from it such as razors, bangles, axes are mentioned.
One verse mentions purification of ayas. Some scholars believe that ayas
refers to iron and the words dham and karmara refer to iron-welders.

The transition of Vedic society from semi-nomadic life to settled


agriculture in the later Vedic age lead to an increase in trade and
competition for resources. Agriculture dominated the economic activity
along the Ganges valley during this period.

Agricultural operations grew in complexity and usage of iron implements


(krishna–ayas or shyama–ayas, literally black metal or dark metal)
increased.

Q Source: Chapter 3: 11th Standard TamilNadu History Textbook

33. Commercial banks create credit by


1. Lending demand deposits to prospective customers
2. Holding excess reserves with the authorization from RBI
3. Requesting RBI to print currency for meeting liquidity crunch

Select the correct answer using the codes below.

a) 1 only
b) 2 and 3 only
c) 1 and 3 only
d) 3 only

Solution: a)

Justification: Statement 1: Credit creation is the process by which the


money supply of an economy is increased.

Commercial bank lending also creates money in the form of demand


deposits. Through the fractional reserve banking (explained in NCERT),


INSIGHTS PRELIMS TEST SERIES 2017

www.insightsonindia.com www.insightsias.com
TEST – 13 Solutions

bank lending multiplies the amount of broad money beyond the amount
of base money originally created by the central bank.

Statement 2: Holding excess reserves will only reduce the loanable


funds with the banks, so 2 is wrong.

Statement 3: Banks are not authorized to submit such requests to RBI.


Moreover, it is not simply the printing of currency which generates more
money, but its repeated lending under the fractional reserve system.

Q Source: Chapter 4: 12th NCERT Macroeconomics

34. Some of the major features of the Chalcolithic period is\are


that
1. People started using copper and bronze metals.
2. A new technology for smelting metal ore was developed.
3. Painted pottery was practiced during this period.

Select the correct answer using the codes below.

a) 1 and 2 only
b) 2 and 3 only
c) 3 only
d) 1, 2 and 3

Solution: d)

Justification: Statement 1: It is marked by the use of copper as the


first major metal in India. Bronze too was used. The Chalcolithic age is
followed by Iron Age. Iron is frequently referred to in the Vedas.

Statement 2: The early period of the age is characterized by the


widespread use of iron or steel. The adoption of these materials
coincided with other changes in society, including differing agricultural
practices, religious beliefs and artistic styles.

Statement 3: Black on Red ware is a famous pottery of this age.

Q Source: Page 16: 11th Standard TamilNadu History Textbook


INSIGHTS PRELIMS TEST SERIES 2017

www.insightsonindia.com www.insightsias.com
TEST – 13 Solutions

35. Generally, an economy is considered ‘open’ if


1. Its foreign trade as a proportion of GDP is quite high
2. It is a fast growing economy
3. It has a stable and large financial system

Select the correct answer using the codes below.

a) 1 and 2 only
b) 2 and 3 only
c) 1 only
d) 1 and 3 only

Solution: c)

Justification: Statement 1: Total foreign trade (exports + imports) as a


proportion of GDP is a common measure of the degree of openness of an
economy.

An open economy trades with other nations in goods, services and at


times in financial assets.

• First, when a nation buys foreign goods, this spending escapes as a


leakage from the circular flow of income decreasing aggregate
demand.

• Second, our exports to foreigners enter as an injection into the


circular flow, increasing aggregate demand for domestically
produced goods.

Statement 2: Even a closed economy can be a fast growing economy. So,


2 is wrong.

Statement 3: Number or depth of financial institutions does not show


how open an economy is. However, with more openness, one can expect
that FIs grow large in number and have deeper penetration of the
economic system.


INSIGHTS PRELIMS TEST SERIES 2017

www.insightsonindia.com www.insightsias.com
TEST – 13 Solutions

Q Source: Page 76: 12th Macroeconomics NCERT

36. Why does monetary policy fail to be effective when an


economy undergoes liquidity trap?
1. Interest rates do not fall below a certain limit.
2. Increase in money supply is held as speculative balance by
people.

Which of the above is/are correct?

a) 1 only
b) 2 only
c) Both 1 and 2
d) None

Solution: c)

Justification: A liquidity trap occurs when low / zero interest rates fail
to stimulate consumer spending and monetary policy becomes
ineffective.

Statement 1: In such as situation, an increase in the money supply could


fail to increase spending because interest rates can't fall further. This is
because a liquidity trap means consumers' preference for liquid assets
(cash) is greater than the rate at which the quantity of money is growing.

Statement 2: New money supply by central bank does not go into


investment and asset creation. Instead, people hoard the new money
supply to speculate in bond market. Since interest rates are already low,
everyone believes they will go and they will suffer a capital loss on bonds.
So, all new money ends up as idle speculative balances with people (in
anticipation of interest rates rise). Monetary policy fails.

So any attempt by policymakers to get individuals to hold non-liquid


assets in the form of consumption by increasing the money supply won't
work.


INSIGHTS PRELIMS TEST SERIES 2017

www.insightsonindia.com www.insightsias.com
TEST – 13 Solutions

Q Source: Page 39: 12th Macroeconomics NCERT

37. Long-term economic growth can be achieved by


a) Neglecting capital goods and focussing fully on consumption
goods
b) Neglecting consumer goods and focussing fully on consumption
goods
c) Maintaining a judicious balance between capital goods and
consumption goods
d) Any of the above

Solution: c)

Justification: Capital goods make production of consumption goods


(like cars) possible.

Since resources are limited in an economy, they can either be


channelized for immediate consumption or immediate investment to
feed long-term consumption.

If they are channelized for immediate consumption (i.e. say we are


producing more cars but not car making machines), a time would come
when demand would overshoot supply and economic stagnation would
follow.

If however, a balance is maintained by sacrificing some consumption


goods to produce more capital goods, we can have more aerated drinks
in the future as there will be more machines to fulfil demand.

Only this generates long-term growth. This is why Raghuram G Rajan,


former RBI Governor, prescribed moving from consumption spending to
investment spending for the Indian economy, so that a judicious can be
achieved.

Q Source: Page 12: 12th Macroeconomics NCERT


INSIGHTS PRELIMS TEST SERIES 2017

www.insightsonindia.com www.insightsias.com
TEST – 13 Solutions

38. The system of ‘dyarchy’ introduced by the Government of


India Act of 1919 meant that
a) Both Centre and provincial legislatures had the power of
legislate in their own spheres.
b) Both the British Parliament and the Central Legislature had the
right to make laws for India
c) Certain subjects were devolved to the ‘Indian’ members of the
Viceroy’s executive council and certain others to the ‘British’
members of the executive council
d) None of the above

Solution: d)

Learning: The 1919 Act relaxed the central control over the provinces
by demarcating and separating the central and provincial subjects.

• The central and provincial legislatures were authorised to make


laws on their respective list of subjects. However, the structure of
government continued to be centralised and unitary.

• It further divided the provincial subjects into two parts—


transferred and reserved. The transferred subjects were to be
administered by the governor with the aid of ministers responsible
to the legislative Council.

• The reserved subjects, on the other hand, were to be administered


by the governor and his executive council without being
responsible to the legislative Council.

• This dual scheme of governance was known as ‘dyarchy’—a term


derived from the Greek word di-arche which means double rule.
However, this experiment was largely unsuccessful.

Q Source: Chapter 1: Indian Polity: M Laxmikanth


INSIGHTS PRELIMS TEST SERIES 2017

www.insightsonindia.com www.insightsias.com
TEST – 13 Solutions

39. The popular JVP committee was setup to address


a) Integration of princely states with India
b) Reorganization of states on linguistic basis
c) Reservation concerns for minority communities
d) Regressive religious provisions in colonial Indian laws

Solution: b)

Learning: The integration of princely states with the rest of India has
purely an ad hoc arrangement. There was a demand from different
regions, particularly South India, for reorganisation of states on
linguistic basis.

• Accordingly, in June 1948, the Government of India appointed the


Linguistic Provinces Commission under the chairmanship of S K
Dhar to examine the feasibility of this.

• The commission submitted its report in December 1948 and


recommended the reorganisation of states on the basis of
administrative convenience rather than linguistic factor.

• This created much resentment and led to the appointment of


another Linguistic Provinces Committee by the Congress in
December 1948 itself to examine the whole question afresh.

• It consisted of Jawaharlal Nehru, Vallahbhai Patel and Pattabhi


Sitaramayya and hence, was popularly known as JVP Committee.

• It submitted its report in April 1949 and formally rejected language


as the basis for reorganisation of states.

• However, in October 1953, the Government of India was forced to


create the first linguistic state, known as Andhra state, by
separating the Telugu speaking areas from the Madras state.

Q Source: Chapter 5: Indian Polity: M Laxmikanth


INSIGHTS PRELIMS TEST SERIES 2017

www.insightsonindia.com www.insightsias.com
TEST – 13 Solutions

40. The real exchange rate (RER) is often taken as a measure of a


country’s international competitiveness because
1. It is not subject to depreciation by destabilizing speculation.
2. It takes into account purchasing power of nations involved.
3. It is fixed by an agreement between the Central banks involved.

Select the correct answer using the codes below.

a) 1 and 2 only
b) 1 only
c) 2 and 3 only
d) 2 only

Solution: d)

Justification: Statement 2: The real exchange rate is often taken as a


measure of a country’s international competitiveness as it takes into
account purchasing power at both nations.

The real exchange rates are nothing but the nominal exchange rates
multiplied by the price indices of the two countries.

This means the market price level of goods and services, given by indices
of inflation. So if the price level in the US is higher than the price level in
India, then the real exchange rate of the rupee versus the dollar will be
greater than the nominal exchange rate.

Suppose the nominal exchange rate is Rs 50 and US prices are greater


than Indian prices, a dollar will buy more in India than what Rs 50 will
buy in the US.

Statement 1: Just like NER, RER too is subject to devaluations and


depreciation. RER is only a mathematical adjustment of NER. If NER is
volatile, RER too will be volatile. So, 1 is incorrect.

Statement 3: Since NER is not fixed by an agreement between Central


banks, RER too is not. 3 will be incorrect.

Q Source: Page 79: 12th Macroeconomics NCERT


INSIGHTS PRELIMS TEST SERIES 2017

www.insightsonindia.com www.insightsias.com
TEST – 13 Solutions

41.Fiscal consolidation as mandated in the FRBM Act was put on hold


in 2007-08 in view of
a) High fiscal deficit
b) Increasing interest payments
c) High GDP forecasts
d) Global recession

Solution: d)

Justification: The implementation of FRBM Act/FRLs improved the


fiscal performance of both centre and states. The States had achieved the
targets much ahead the prescribed timeline. The Centre was on the path
of achieving this objective right in time.

However, due to the global financial crisis (and impending weak


demand), this was suspended and the fiscal consolidation as mandated
in the FRBM Act was put on hold in 2007-08.

The crisis period called for increase in expenditure by the government to


boost demand in the economy.

As a result of fiscal stimulus, the government has moved away from the
path of fiscal consolidation.

However, it should be noted that strict adherence to the path of fiscal


consolidation during pre crisis period created enough fiscal space for
pursuing counter cyclical fiscal policy.

Q Source: Page 64: 12th Macroeconomics NCERT

42. The Pradhan Mantri Garib Kalyan Yojana (PMGKY) deals


with
a) Disclosure of unaccounted wealth
b) Formation of farmer clubs in rural areas
c) Development of model villages in backward areas


INSIGHTS PRELIMS TEST SERIES 2017

www.insightsonindia.com www.insightsias.com
TEST – 13 Solutions

d) Provision of health, education and sanitation facilities to BPL


families

Solution: a)

Learning: The scheme was notified along with other provisions of


Taxation Laws (Second Amendment) Act, 2016 came into effect from
2016.

PMGKY is Union Government’s second income disclosure scheme (IDS)


to allow tax evaders to come clean with unaccounted wealth.

It provides for 50 per cent tax and surcharge on declarations of


unaccounted cash deposited in banks.

Besides, declarations made under it will be kept confidential and shall


not be admissible as evidence under any Act (ex. Wealth-tax Act, Central
Excise Act, Companies Act etc.).

However, declarant will have no immunity under particular Criminal


Acts of the Scheme.

The money or revenue generated from disclosure of unaccounted cash


will be used for welfare schemes for the poor. It will be mainly used for
projects in irrigation, infrastructure, primary education, primary health,
housing, toilets and livelihood so that there is justice and equality.

Q Source: PIB: Major current schemes

43. The boundary commission headed by Radcliff demarcated


a) Provincial areas of Bihar and Assam post Bengal Partition 1905
b) Boundaries between the two Dominions of India and Pakistan
c) Burmese Enclaves in the north-eastern frontier of India
d) North-west frontier of India post-Soviet invasion

Solution: b)


INSIGHTS PRELIMS TEST SERIES 2017

www.insightsonindia.com www.insightsias.com
TEST – 13 Solutions

Learning: The Radcliffe Line was published on 17 August 1947 as a


boundary demarcation line between India and Pakistan.

Pakistan included the provinces of West Punjab, Sind, Baluchistan, East


Bengal, North-Western Frontier Province and the district of Sylhet in
Assam.

The help of a referendum was taken to establish the total geographical


area of Pakistan. The referendum in the North-Western Frontier
Province and Sylhet was in favour of Pakistan.

Q Source: Chapter 2: Indian Polity: M Laxmikanth

44. Consider the following sources of the Constitution and the


features borrowed from them.
1. Canadian Constitution: Parliamentary privileges and
bicameralism
2. Australian Constitution: Concurrent List and joint sitting of
Parliament
3. Weimar Constitution of Germany: Fundamental duties
4. Irish Constitution: Directive Principles of State Policy

Select the correct answer using the codes below.

a) 1, 2 and 4 only
b) 2 and 3 only
c) 1, 3 and 4 only
d) 2 and 4 only

Solution: d)

Justification: Statement 1: They were borrowed from British


constitution.

Statement 3: They were borrowed from Soviet Constitution.

This is an exhaustive list of major features.


INSIGHTS PRELIMS TEST SERIES 2017

www.insightsonindia.com www.insightsias.com
TEST – 13 Solutions

Q Source: Chapter 2: Indian Polity: M Laxmikanth

45. Public finance largely deals with the allocative, distributive


and stabilizing functions. Consider the following arguments about
them.
1. Allocative function may affect the distributive function.
2. Stabilizing function may have a bearing on allocative function.

Which of the above is/are correct?

a) 1 only
b) 2 only
c) Both 1 and 2
d) None

Solution: c)


INSIGHTS PRELIMS TEST SERIES 2017

www.insightsonindia.com www.insightsias.com
TEST – 13 Solutions

Justification: Statement 1: Coordination between different budget


functions is a tricky exercise. It is because these often overlap in practice.

Suppose first that the public wishes an increased supply of public


services (ALLOCATIVE FUNCTION) which is to be financed by
increased taxes (DISTRIBUTE FUNCTION). This leads in turn, to the
question of how they taxes should be distributed as it will change the
distribution of income.

Suppose that society wishes to shift distribution in the direction of


greater (lesser) equality. Such a shift may be accomplished by using
progressive (regressive) taxes to finance transfers to lower (higher)
incomes. But it may also be done by increasing (reducing) the supply of
public services of particular value to low (high) income groups. This,
however, interferes with the pattern of public services which consumers
want to obtain at a given distribution of income. Once more, one policy
objective may be implemented such that it interferes with another.

Statement 2: Finally, consider the role of fiscal policy in


STABILIZATION. Suppose that a more (less) expansionary policy is
needed. This may be accomplished by raising (lowering) outlays on
public services or by reducing (raising) the level of taxation, altering the
allocation and distribution function.

Q Source: Improvisation: Page 62: 12th Macroeconomics NCERT

46. Consider the following statements.

Assertion (A): Keynesians reject the effectiveness of a fiscal


stimulus in reviving the market.

Reason (R): Keynesians believe that investment depends solely on


the rate of interest.

In the context of the above, which of these is correct?


INSIGHTS PRELIMS TEST SERIES 2017

www.insightsonindia.com www.insightsias.com
TEST – 13 Solutions

a) A is correct, and R is an appropriate explanation of A.


b) A is correct, but R is not an appropriate explanation of A.
c) A is correct, but R is incorrect.
d) Both A and R are incorrect.

Solution: d)
Justification: Keynesian economists often argue that private sector
decisions sometimes lead to inefficient macroeconomic outcomes which
require active policy responses by the public sector, in particular,
monetary policy actions by the central bank and fiscal policy actions by
the government, in order to stabilize output over the business cycle.

Keynesians believe that investment does not depend solely on the rate of
interest but on various factors such as investor’s expectations, market
conditions, return on capital etc. In the absence of state intervention,
investment may fluctuate erratically. This may cause aggregate demand
to go down and reduce economic growth and employment.

So, Keynesian economics advocates a mixed economy – predominantly


private sector, but with a role for government intervention during
recessions.

Q Source: Improvisation: Page 6: 12th Macroeconomics NCERT

47. The major features of parliamentary government in India


is/are?
1. Collective responsibility of the executive to the legislature
2. Leadership of the prime minister
3. Complete separation of the legislature and the executive organs

Select the correct answer using the codes below.

a) 1 and 2 only
b) 2 and 3 only
c) 1 and 3 only


INSIGHTS PRELIMS TEST SERIES 2017

www.insightsonindia.com www.insightsias.com
TEST – 13 Solutions

d) 1, 2 and 3

Solution: a)

Justification: Statement 3: The parliamentary system is based on the


principle of cooperation and co-ordination between the legislative and
executive organs while the presidential system is based on the doctrine
of separation of powers between the two organs. So, 3 is incorrect.

Learning: Other major features are: (a) Presence of nominal and real
executives; (b) Majority party rule, (c) Collective responsibility of the
executive to the legislature, (d) Membership of the ministers in the
legislature, (e) Leadership of the prime minister or the chief minister, (f)
Dissolution of the lower House (Lok Sabha or Assembly).

Q Source: Chapter 3: Indian Polity: M Laxmikanth

48. Inflation generally results from


1. An excess of supply over demand
2. An excess of demand over supply
3. If income increases faster than the increase in productive
capacity of the economy

Select the correct answer using the codes below.

a) 1 and 2 only
b) 2 only
c) 2 and 3 only
d) 1 and 3 only

Solution: c)

Justification: In simple terms, inflation is basically too much money


chasing too few goods, or excess demand chasing limited supply. In both
these cases, the prices of goods rises faster as individual consumers bid
process higher in order to get the good.


INSIGHTS PRELIMS TEST SERIES 2017

www.insightsonindia.com www.insightsias.com
TEST – 13 Solutions

Statement 1: So, 1 is incorrect, as excess supply is likely to bring prices


down, not high.

Statement 2: 2 is explained by the above.

Statement 3: If income rises faster, demand for goods and services will
also rise. On the other hand, if the economy is unable to satisfy the
increased demand, for e.g. due to poor infrastructure, lack of production
etc, the higher income will spiral the prices upwards and lead to high
inflation.

Q Source: Improvisation: Chapter 4: 12th Macroeconomics NCERT

49. ‘Total Factor Productivity (TFP)’ of an economy can improve


in which of these ways?
1. Increasing taxation on the private sector
2. Investing in research and development (R&D)
3. Creating infrastructure

Select the correct answer using the codes below.

a) 1 and 2 only
b) 2 and 3 only
c) 3 only
d) 1, 2 and 3

Solution: b)

Justification: If a nation has the same stock of land, labour and


capital, and yet it achieves high rates of GDP growth consistently. If
inputs are same, how does output increase over time?

Either the individual productivities of factors would have increased, or


the total combined productivity of the economy has increased.

This can happen either by better technology, better infrastructure etc


that improve the total factor productivity of the economy. Suppose, now


INSIGHTS PRELIMS TEST SERIES 2017

www.insightsonindia.com www.insightsias.com
TEST – 13 Solutions

a factory requires 10 hours to produce a good. With better techniques of


production, the same can be achieved in 2 hours and the company can
produce than ever before.

This TFP is called as the part of the output not explained by the change
in inputs.

Q Source: Improvisation: Chapter 2: 12th Macroeconomics NCERT

50. The Constitution of India establishes a federal system of


government. However, which of these are Unitary (Central) biases
in the federal system?
1. A written Constitution
2. Rigidity of Constitution
3. Independent Judiciary
4. All-India services

Select the correct answer using the codes below.

a) 1 and 2 only
b) 3 and 4 only
c) 1 and 4 only
d) 4 only

Solution: d)

Justification: Statement 1: A written constitution actually shows


federal features of a constitution since both units (central and state) are
bound by a common framework.

Statement 2: A rigid constitution is essential to a federal system so that


the Centre cannot unilaterally amend the constitution.

Statement 3: An independent judiciary is crucial to maintain rule of law


and ensure harmonious and unbiased dispute redressal between federal
units.


INSIGHTS PRELIMS TEST SERIES 2017

www.insightsonindia.com www.insightsias.com
TEST – 13 Solutions

Statement 4: The All-India services are controlled jointly by the State


and Centre. Ultimate control lies with the Centre and only immediate
control lies with states. These civil servants severely limit the autonomy
of the federal units. So, 4 is correct.

Other such features are a strong Centre, single Constitution, single


citizenship, flexibility of Constitution, integrated judiciary, appointment
of state governor by the Centre, emergency provisions, and so on.

Q Source: Chapter 3: Indian Polity: M Laxmikanth

51. Which of these bills is/are introduced as a “Secret Bill” every year
in Lok Sabha?
1. Appropriation Bill
2. Finance Bill

Select the correct answer using the codes below.

a) 1 only
b) 2 only
c) Both 1 and 2
d) None

Solution: c)
Justification: Statement 1 and 2: Lok Sabha rules of business say:

No Bill shall be included for introduction in the list of business for a day
until after copies thereof have been made available for the use of
members for at least two days before the day on which the Bill is
proposed to be introduced:

• Provided that Appropriation Bills, Finance Bills and such Secret


Bills as are not put down in the list of business may be introduced
without prior circulation of copies to members:


INSIGHTS PRELIMS TEST SERIES 2017

www.insightsonindia.com www.insightsias.com
TEST – 13 Solutions

• Provided further that in other cases, where the Minister desires


that the Bill may be introduced earlier than two days after the
circulation of copies or even without prior circulation, he shall
give full reasons in a memorandum for the consideration of the
Speaker explaining as to why the Bill is sought to be introduced
without making available to members copies thereof in advance,
and if the Speaker gives permission, the Bill shall be included in
the list of business for the day on which the Bill is proposed to be
introduced

This means that such bills (even other than Finance and appropriation
bills) can be kept as “Secret” if the Minister so desires.

The purpose behind this is to prevent the pre-disclosure of sensitive


financial information, which can be exploited by outsiders to gain unfair
advantage. For e.g. if government has proposed a major tax change on
tobacco products, and the information is known before hand by some
traders, they can speculatively trade in stocks of tobacco companies and
gain unfair advantage.

Q Source: Page 62: 12th Macroeconomics NCERT

52. Consider the following about the Adaptation Fund (AF) used
for financing climate change projects.
1. It was established under the Convention on Biological Diversity
(CBD) in 1992.


INSIGHTS PRELIMS TEST SERIES 2017

www.insightsonindia.com www.insightsias.com
TEST – 13 Solutions

2. It is financed with a share of proceeds from the clean


development mechanism (CDM).
3. It is supervised and managed by the United Nations Sustainable
Development Forum (UNSDF).

Select the correct answer using the codes below.

a) 1 only
b) 2 and 3 only
c) 2 only
d) 1, 2 and 3

Solution: c)
Justification: Statement 1: It was founded in 2001 to finance concrete
adaptation projects and programmes in developing country Parties to
the Kyoto Protocol that are particularly vulnerable to the adverse effects
of climate change.

Statement 2: The Adaptation Fund is financed with a share of proceeds


from the clean development mechanism (CDM) project activities and
other sources of funding. The share of proceeds amounts to 2 per cent of
certified emission reductions (CERs) issued for a CDM project activity.

Statement 3: The Adaptation Fund is supervised and managed by the


Adaptation Fund Board (AFB). The AFB is composed of 16 members
and 16 alternates and meets at least twice a year (Membership of the
AFB).

Q Source:
http://unfccc.int/cooperation_and_support/financial_mechanism/ada
ptation_fund/items/3659.php


INSIGHTS PRELIMS TEST SERIES 2017

www.insightsonindia.com www.insightsias.com
TEST – 13 Solutions

53. Consider the following about the System of Rice


Intensification (SRI) cultivation.
1. SRI involves higher water use than traditional techniques of rice
cultivation.
2. SRI requires higher plant density to ensure optimum nutrient
utilization of soil.

Which of the above is/are correct?

a) 1 only
b) 2 only
c) Both 1 and 2
d) None

Solution: d)
Justification: SRI supports reduced and controlled water application
as one of its principles. So, 1 is wrong.

Other principles are early, quick and healthy plant establishment


(transplantation); reduced plant density and improved soil conditions
through enrichment with organic matter. So, 2 is wrong.

Based on these principles, farmers can adapt recommended SRI


practices to respond to their agroecological and socioeconomic
conditions.

SRI principles and practices have been adapted for rainfed rice as well as
for other crops (such as wheat, sugarcane and tef, among others), with
yield increases and associated economic benefits.

Q Source:
http://www.agritech.tnau.ac.in/agriculture/agri_cropproduction_rice_s
ri.html

54. Consider the following about the Charter Act of 1833 which
was an important step towards centralisation and administrative
reforms in British India.


INSIGHTS PRELIMS TEST SERIES 2017

www.insightsonindia.com www.insightsias.com
TEST – 13 Solutions

1. It deprived the Governor-General of India of his legislative


powers and vested it in a nominated assembly.
2. It provided for reservation of Indians in the covenanted civil
services.

Which of the above is/are correct?

a) 1 only
b) 2 only
c) Both 1 and 2
d) None

Solution: d)
Justification: Statement 1: It made the Governor-General of Bengal as
the Governor-General of India and vested in him all civil and military
powers. Thus, the act created, for the first time, a Government of India
having authority over the entire territorial area possessed by the British
in India.

It thus deprived the governor of Bombay and Madras of their legislative


powers. The Governor-General of India was given exclusive legislative
powers for the entire British India.

Statement 2: The Charter Act of 1833 attempted to introduce a system of


open competition for selection of civil servants, and stated that the
Indians should not be debarred from holding any place, office and
employment under the Company. However, this provision was negated
after opposition from the Court of Directors.

Learning: It ended the activities of the East India Company as a


commercial body, which became a purely administrative body. It
provided that the company’s territories in India were held by it ‘in trust
for His Majesty, His heirs and successors’.

Q Source: Chapter 1: Indian Polity: M Laxmikanth


INSIGHTS PRELIMS TEST SERIES 2017

www.insightsonindia.com www.insightsias.com
TEST – 13 Solutions

55. Consider the following with reference to the Marker assisted


selection techniques.

Assertion (A): It is more efficient for breeding as compared to


conventional breeding techniques.

Reason (R): In this technique, a trait of interest is selected based


on a “marker” linked to a “trait of interest” rather than on the
“trait” itself.

In the context of the above, which of these is correct?

a) A is correct, and R is an appropriate explanation of A.


b) A is correct, but R is not an appropriate explanation of A.
c) A is correct, but R is incorrect.
d) Both A and R are incorrect.

Solution: a)
Justification: Marker aided selection (MAS) is an indirect selection
process where a trait of interest is selected based on a marker
(morphological, biochemical or DNA/RNA variation) linked to a trait of
interest (e.g. productivity, disease resistance, abiotic stress tolerance,
and quality), rather than .

Marker-assisted selection may greatly increase the efficiency and


effectiveness for breeding compared to conventional breeding. The
fundamental advantages of MAS compared to conventional phenotypic
selection are:

• Simpler compared to phenotypic screening

• Selection may be carried out at seedling stage

• Single plants may be selected with high reliability.

These advantages may accelerate the line development in breeding


programs.

You can read more here


http://www.knowledgebank.irri.org/ricebreedingcourse/Marker_assiste
d_breeding.htm


INSIGHTS PRELIMS TEST SERIES 2017

www.insightsonindia.com www.insightsias.com
TEST – 13 Solutions

Q Source: Agritech TNAU: Breeding programmes

56. In the year 2016, the Supreme Court and High Courts have
declared some reservation schemes of Central and State
governments, such as Gujarat and Haryana, as void. Which of the
following fundamental arguments have been given by the courts in
these judgments?
1. Reservation based on economic backwardness alone is
unconstitutional.
2. Reservation based solely on grounds of caste is untenable and
unjustified.

Which of the above is/are correct?

a) 1 only
b) 2 only
c) Both 1 and 2
d) None

Solution: c)
Justification: Even before these judgments, the court had declared in
the famous Mandal case (1992) that EBCs cannot be given reservations.

The recent judgements only improvise on the fundamental arguments


raised in that case.

Statement 1: The Gujarat High Court in 2016 set aside the Gujarat State
government’s ordinance granting 10 per cent reservation in education
and employment for Economically Backward Classes (EBCs).

The court said the provision of quota for non-reserved EBCs, over and
above the existing 49 per cent quota for reserved categories, would take
the reservation percentage to well over the Supreme Court-mandated
limit of 50. Moreover, such reservation was unconstitutional.


INSIGHTS PRELIMS TEST SERIES 2017

www.insightsonindia.com www.insightsias.com
TEST – 13 Solutions

Statement 2: Supreme Court ruled that “caste” and “historical injustice”


cannot blind a state in according backward status to a community and
that new emerging groups such as transgenders must be identified for
quota benefits.

In this view, it scrapped the Centre’s March 2014 notification to include


Jats in the central list of the Other Backward Classes (OBC) category in
nine states.

Q Source: http://indianexpress.com/article/india/india-others/sc-
quashes-decision-to-include-jats-in-obc-category-rules-caste-cant-be-
sole-ground/

http://www.thehindubusinessline.com/news/national/gujarat-high-
court-quashes-10-quota-for-ebc/article8942278.ece

57. Net foreign factor income (NFIA) of India will change if


1. More remittances are received from Indian workers in Gulf
countries
2. The returns of foreign investors on investment in India start
rising

Which of the above is/are correct?

a) 1 only
b) 2 only
c) Both 1 and 2
d) None

Solution: c)
Justification: NFIA is the difference between the aggregate amount
that a country’s citizens and companies earn abroad, and the aggregate
amount that foreign citizens and overseas companies earn in that
country.


INSIGHTS PRELIMS TEST SERIES 2017

www.insightsonindia.com www.insightsias.com
TEST – 13 Solutions

The NFIA level is generally not substantial in most nations, since factor
payments earned by their citizens and those paid to foreigners more or
less offset each other.

Statement 1: If gulf workers send more remittances, India’s NFIA will


rise. This is because labour is a ‘factor’ of production which has been
deployed in a foreign nation.

Statement 2: Capital (machinery, investment) is another factor of


production. The income coming from this factor becomes a part of NFIA,
if either foreign investors invest in India or Indian investors invest
abroad.

Q Source: Page 26: 12th Macroeconomics NCERT

58. State legislatures can make laws to enforce


1. Fundamental Rights
2. Directive Principles of State Policy
3. Fundamental Duties

Select the correct answer using the codes below.

a) 1 and 2 only
b) 2 and 3 only
c) 1 and 3 only
d) 1, 2 and 3

Solution: b)

Justification: Statement 1: Most of them are directly enforceable (self-


executed) while a few of them can be enforced on the basis of a law made
for giving effect to them.

Such a law can be made only by the Parliament and not by state
legislatures so that uniformity throughout the country is maintained
(Article 35). So, 1 is incorrect.


INSIGHTS PRELIMS TEST SERIES 2017

www.insightsonindia.com www.insightsias.com
TEST – 13 Solutions

Statement 2: For e.g. Tamilnadu government came up with many


schemes to universalize food distribution, health care, education etc
which fulfil provisions of DPSP. So, 2 is correct.

Statement 3: For e.g. environmental conservation and forests fall in


concurrent lists, the preservation of which also falls under fundamental
duties. State legislatures can make laws on these matters, so, 3 is correct.

Q Source: Chapter 7: Indian Polity: M Laxmikanth

59. If the government constantly runs a budget surplus, it may


lead to contraction of
1. Aggregate demand in the economy
2. Money supply in the economy

Which of the above is/are correct?

a) 1 only
b) 2 only
c) Both 1 and 2
d) None

Solution: a)

Justification: Statement 1: Government spending is an important part


of aggregate demand or output (Y= C+ I + G), where investment and
consumption are other components. For e.g. AD was revived in India in
the wake of 2008 recession by a fiscal stimulus.

Statement 2: Money supply is regulated by the monetary authorities,


and the amount of currency in circulation is controlled by the RBI. Even
if the government runs a budget surplus, it only means that some other
entity is using the surplus money resources. Money supply does not
increase or decrease by fiscal policy actions.

Q Source: Chapter 5: 12th NCERT Macroeconomics


INSIGHTS PRELIMS TEST SERIES 2017

www.insightsonindia.com www.insightsias.com
TEST – 13 Solutions

60. Consider the following about the religious practices of the


Harappan civilization.
1. Linga worship was prevalent during their age.
2. Religion was strictly masculine and patriarchal in nature.

Which of the above is/are correct?

a) 1 only
b) 2 only
c) Both 1 and 2
d) None

Solution: a)
Justification: Statement 1: The chief male deity was Pasupati, (proto-
Siva) represented in seals as sitting in a yogic posture with three faces
and two horns. In latter times, Linga worship was prevalent.

Statement 2: The chief female deity was the Mother Goddess, which
shows that the religion was not strictly masculine in nature. She was
represented in terracotta figurines. So, 2 is wrong.

Moreover, natural entities like trees and animals were worshipped by the
Harappans. They also believed in ghosts and evil forces and used
amulets as protection against them.

Q Source: Page 23: 11th Standard TamilNadu History Textbook

61.Article 14 of the Constitution forbids class legislation. What is


implied by this?
1. Laws that discriminate between people based on unreasonable
classification stand void.


INSIGHTS PRELIMS TEST SERIES 2017

www.insightsonindia.com www.insightsias.com
TEST – 13 Solutions

2. Class legislation applies only to private individuals.

Which of the above is/are correct?

a) 1 only
b) 2 only
c) Both 1 and 2
d) None

Solution: a)
Justification: Statement 2: Article 14 says that the State shall not deny
to any person equality before the law or the equal protection of the laws
within the territory of India. This provision confers rights on all persons
whether citizens or foreigners. Moreover, the word ‘person’ includes
legal persons, viz, statutory corporations, companies, registered societies
or any other type of legal person. So, 2 is wrong.

Statement 1: The Supreme Court held that where equals and unequals
are treated differently, Article 14 does not apply. While Article 14 forbids
class legislation, it permits reasonable classification of persons, subjects
and transactions by the law. But the classification should not be
arbitrary, artificial or evasive. Rather, it should be based on an
intelligible differential and substantial distinction.

For e.g. a special law can be made for transgender as this classification is
reasonable and just, but classification made solely on grounds of
financial or economic status may be challenged in the courts for being
based in unreasonable classification.

Q Source: Chapter 7: Indian Polity: M Laxmikanth

62. RBI publishes figures for alternative measures of money


supply, such as M0, M1, M2 and M3. This classification is based on
a) Difference in Liquidity of these measures
b) The availability of these monetary resources in the economy


INSIGHTS PRELIMS TEST SERIES 2017

www.insightsonindia.com www.insightsias.com
TEST – 13 Solutions

c) Partial or full convertibility of these measures of money supply


d) None of the above

Solution: a)
Justification: Reserve Money (M0) is “High Powered Money” or
“Monetary Base”. Reserve Money forms the basis for the creation of
liquid money in the economy. It includes currency with the Public; Other
Deposits with the RBI; Cash Reserves of the banks held with themselves
and Cash Reserves of the Banks held with RBI.

M1 = Currency with the public + Demand Deposits of public in Banks

When a third component viz. Post office Savings Deposits is also added
to M1, it becomes M2. M2 = M1 + Post Office Savings.

M3 is the most commonly used measure of money supply. It is also


known as aggregate monetary resources. M3 = Narrow money + Time
Deposits of public with banks

Out of all, M3 is the least liquid.

All the money held with public, RBI as well as government is called Total
Stock of Money. Money Supply is that part of this Total Stock of Money
which is with public. By public we refer to the households, firms, local
authorities, companies etc.

Thus, public money does not include the money held by the government
and the money held as CRR with RBI and SLR with themselves by
commercial banks.

The reason of excluding the above two categories from money supply is
that this money held by the Government and RBI is out of circulation.

Q Source: Page 40: 12th Macroeconomics NCERT

63. Special provisions (under Part XXI) of the Constitution are


applicable to the States of


INSIGHTS PRELIMS TEST SERIES 2017

www.insightsonindia.com www.insightsias.com
TEST – 13 Solutions

1. Maharashtra
2. Gujarat
3. Jharkhand
4. Chattisgarh
5. Andhra Pradesh

Select the correct answer using the codes below.

a) 1 and 2 only
b) 3 and 4 only
c) 1, 2 and 5 only
d) 1, 2, 3, 4 and 5

Solution: c)
Justification: The provisions of the Constitution pertaining to the
states are applicable to all the states (except Jammu and Kashmir) in the
same manner.

However, the special provisions (under Part XXI) applicable to the States
of Maharashtra, Gujarat, Nagaland, Assam, Manipur, Andhra Pradesh,
Sikkim, Mizoram, Arunanchal Pradesh and Goa override the general
provisions relating to the states as a class.

Statement 2 and 3: But, the provisions for Jharkhand and Chattisgarh


are found only in the 5th Schedule (tribal areas). Further, the Sixth
Schedules contain separate provisions with respect to the administration
of scheduled areas and tribal areas within the NE states.

Q Source: Chapter 5: Indian Polity: M Laxmikanth

64. Which of the following legumes are most commonly used as


green manuring crops?
1. Jowar
2. Mung
3. Guar


INSIGHTS PRELIMS TEST SERIES 2017

www.insightsonindia.com www.insightsias.com
TEST – 13 Solutions

4. Sunflower

Select the correct answer using the codes below.

a) 1 and 2 only
b) 2 and 3 only
c) 1 and 4 only
d) 2 and 4 only

Solution: b)
Justification: Crops suitable for green manuring are divided into two
groups:

• Non-legumes or Non-leguminous crops: The non-legumes used as


green manuring crops provide only organic matter to the soil. The
non-legumes are used for green manuring to a limited extent.

o Examples: Mustard, Wheat, Carrot, Jowar, Maize, Sunflower etc.


So, 1 and 4 are incorrect.

• Legumes or Leguminous Crops: The legumes used as green manuring


crops provide nitrogen as well as organic matter to the soils. Legumes
have the ability of acquiring nitrogen from the air with the help of its
nodule bacteria. The legumes are most commonly used as green
manuring crops.

o Examples: Sannhemp, Djainach, Mung, Cowpea, Lentil, Senji,


Berseem, Guar etc. So, 2 and 3 are correct.

Q Source: Agritech TNAU: Green manure

65. RBI plays an important role as ‘lender of last resort’ in the


Indian Financial system. What do you understand by this role of
RBI?


INSIGHTS PRELIMS TEST SERIES 2017

www.insightsonindia.com www.insightsias.com
TEST – 13 Solutions

1. RBI lends borrowers who could not avail credit through formal
banking channels.
2. RBI can help banks in matters of insolvency and defaults.

Which of the above is/are correct?

a) 1 only
b) 2 only
c) Both 1 and 2
d) None

Solution: b)
Justification: Statement 1: The total amount of deposits held by all
commercial banks in the country is much larger than the total size of
their reserves.

So, if all the account-holders of all commercial banks in the country want
their deposits back at the same time, the banks will not have enough
means to satisfy the need of every accountholder and there will be bank
failures.

The Reserve Bank of India plays a crucial role here. In case of a crisis like
the above it stands by the commercial banks as a guarantor and extends
loans to ensure the solvency of the latter.

Statement 2: This system of guarantee assures individual account-


holders that their banks will be able to pay their money back in case of a
crisis and there is no need to panic thus avoiding bank runs. But, this is
not made as the last resort for individual borrowers, and so statement 2
is wrong.

Q Source: Page 45: 12th Macroeconomics NCERT

66. The responsibility to operate the Special Climate Change


Fund (SCCF) has been entrusted to
a) Global Environment Facility (GEF)


INSIGHTS PRELIMS TEST SERIES 2017

www.insightsonindia.com www.insightsias.com
TEST – 13 Solutions

b) United Nations Environment Programme (UNEP)


c) UN High Level Forum for Sustainable Development
d) None of the above

Solution: a)
Learning: The Special Climate Change Fund (SCCF) was established
under the UNFCCC Convention in 2001 to finance projects relating to:
adaptation; technology transfer and capacity building; energy, transport,
industry, agriculture, forestry and waste management; and economic
diversification.

This fund should complement other funding mechanisms for the


implementation of the Convention.

The Global Environment Facility (GEF), as an operating entity of the


Financial Mechanism, has been entrusted to operate the SCCF.

Q Source:
http://unfccc.int/cooperation_and_support/financial_mechanism/spec
ial_climate_change_fund/items/3657.php

67. Revenue expenditure of the government consists of


1. Interest payments on debt incurred by the government
2. Grants given to state governments
3. Administrative expenditures

Select the correct answer using the codes below.

a) 1 and 2 only
b) 2 and 3 only
c) 1 and 3 only
d) 1, 2 and 3

Solution: d)
Learning: Revenue expenditure consists of all those expenditures of the
government which do not result in creation of physical or financial
assets.


INSIGHTS PRELIMS TEST SERIES 2017

www.insightsonindia.com www.insightsias.com
TEST – 13 Solutions

It relates to those expenses incurred for the normal functioning of the


government departments and various services, interest payments on
debt incurred by the government, and grants given to state governments
and other parties (even though some of the grants may be meant for
creation of assets).

Q Source: Page 64: 12th Macroeconomics NCERT

68. NAZCA is a global platform for climate change action


participation mainly by
a) Governments of Developed nations
b) Least Developed Nations (LDCs)
c) Non-state actors
d) Local governments

Solution: c)
Learning: NAZCA stands for Non-State Actor Zone for Climate Action.

• NAZCA is a global platform that brings together the commitments


to action by companies, cities, subnational regions, investors and
civil society organizations to address climate change.

• NAZCA aims to track the mobilization and action that are helping
countries achieve and exceed their national commitments to
address climate change.

• NAZCA was launched in COP20, alongside the Lima Paris Action


Agenda (LPAA) in 2014.

• It was a central tool for the LPAA to build momentum and support
the adoption of the universal climate agreement at COP21.

• This culminated with its inclusion in the Paris outcome where


countries welcome the efforts of these actors to scale up their
climate actions and encourages the registration of these actions on
NAZCA.

Q Source: UNFCCC Home page


INSIGHTS PRELIMS TEST SERIES 2017

www.insightsonindia.com www.insightsias.com
TEST – 13 Solutions

69. Supreme Court has been vested with ‘original’ and ‘wide’
powers for defending the fundamental rights of citizens. What do
you understand by original powers of SC?
1. SC holds this power exclusively.
2. Citizens may approach the SC directly without necessarily going
through appeals from lower courts.

Which of the above is/are correct?

a) 1 only
b) 2 only
c) Both 1 and 2
d) None

Solution: b)
Justification: Supreme Court has been constituted as the defender and
guarantor of the fundamental rights of the citizens. It has been vested
with the ‘original’ and ‘wide’ powers for that purpose.

• Original, because an aggrieved citizen can directly go to the


Supreme Court, not necessarily by way of appeal.
• Wide, because its power is not restricted to issuing of orders or
directions but also writs of all kinds.

Statement 1: In case of the enforcement of Fundamental Rights, the


jurisdiction of the Supreme Court is original but not exclusive. It is
concurrent with the jurisdiction of the high court under Article 226.

It vests original powers in the high court to issue directions, orders and
writs of all kinds for the enforcement of the Fundamental Rights. It
means when the Fundamental Rights of a citizen are violated, the
aggrieved party has the option of moving either the high court or the
Supreme Court directly.

Q Source: Chapter 7: Indian Polity: M Laxmikanth


INSIGHTS PRELIMS TEST SERIES 2017

www.insightsonindia.com www.insightsias.com
TEST – 13 Solutions

70. The Balance of Payments (BoP) consist of which of the


following items?
1. Annual FDI and FII received
2. Imports and exports
3. Any borrowing of the Government of India from abroad

Select the correct answer using the codes below.

a) 1 and 3 only
b) 2 only
c) 1 only
d) 1, 2 and 3

Solution: d)
Justification: BoP consists of current account and capital account.

Current account includes trade, invisibles, remittances etc.

Capital account includes borrowings and long-term investments/debts of


residents/foreigners with India.

Capital inflows like FDI, FII help bridge the trade deficit and neutralize
BoP.

Q Source: Chapter 6: 12th Macroeconomics NCERT

71. In the annual budget documents of the Government of India,


‘Primary Deficit’ refers to
a) Difference between fiscal deficit of the current year and interest
payments on the previous borrowings


INSIGHTS PRELIMS TEST SERIES 2017

www.insightsonindia.com www.insightsias.com
TEST – 13 Solutions

b) Difference between revenue deficit of the current year and


grants for capital creation
c) Difference between revenue deficit of the present financial year
and grants to states and local bodies
d) Difference between budgetary deficit and capital deficit of the
present financial year

Solution: a)

Learning: Primary Deficit = Fiscal Deficit – Interest Payments

The total borrowing requirement of the government includes the interest


commitments on accumulated debts.

Primary deficit reflects the extent to which such interest commitments


have compelled the government to borrow in the current period.

A shrinking primary deficit indicates progress towards fiscal health. The


Budget document also mentions deficit as a percentage of GDP. This is to
facilitate comparison and also get a proper perspective. Prudent fiscal
management requires that the government does not borrow to consume
in the normal course.

Q Source: Page 65: 12th Macroeconomics NCERT

72. Right to freedom granted under Articles 19 to 22 cover which


of these rights?
1. Right to freedom of speech, movement and expression
2. Protection against arrest and detention in certain cases
3. Prohibition of traffic in human beings
4. Right to freedom against bonded labour

Select the correct answer using the codes below.

a) 1 and 2 only
b) 1, 2 and 4 only
c) 2, 3 and 4 only


INSIGHTS PRELIMS TEST SERIES 2017

www.insightsonindia.com www.insightsias.com
TEST – 13 Solutions

d) 1, 2, 3 and 4

Solution: a)

Justification: It consists of

• Protection of six rights regarding freedom of: (i) speech and


expression, (ii) assembly, (iii) association, (iv) movement, (v)
residence, and (vi) profession (Article 19).
• Protection in respect of conviction for offences (Article 20).
• Protection of life and personal liberty (Article 21).
• Right to elementary education (Article 21A).
• Protection against arrest and detention in certain cases (Article
22).

Right against exploitation (Articles 23–24) cover

• Prohibition of traffic in human beings and forced labour (Article


23).
• Prohibition of employment of children in factories, etc. (Article
24).

Q Source: Chapter 7: Indian Polity: M Laxmikanth

73. Consider the following about the Clean Development


Mechanism (CDM).
1. CDM allows industrialized countries to buy certified emission
reduction (CER) and to invest in emission reductions in
developing economies.
2. “Additionality” is ignored as an assessment factor for qualifying
projects under CDM.

Which of the above is/are correct?

a) 1 only
b) 2 only
c) Both 1 and 2
d) None


INSIGHTS PRELIMS TEST SERIES 2017

www.insightsonindia.com www.insightsias.com
TEST – 13 Solutions

Solution: a)

Justification: Statement 1: The Clean Development Mechanism


(CDM), defined the Protocol, allows a country with an emission-
reduction or emission-limitation commitment under the Kyoto Protocol
(Annex B Party) to implement an emission-reduction project in
developing countries. Such projects can earn saleable certified emission
reduction (CER) credits, each equivalent to one tonne of CO2, which can
be counted towards meeting Kyoto targets.

Statement 2: If emission reduction in a project would have happened


anyway, it is not counted as part of CDM. In order to maintain the
environmental effectiveness of the Kyoto Protocol, emission savings
from the CDM must be additional (World Bank, 2010). Without
additionality, the CDM amounts to an income transfer to non-Annex I
countries. Additionality is, however, difficult to prove, and is the subject
of vigorous debate.

Assessing additionality is costly and has created delays (bottlenecks) in


approving CDM projects as per World Bank (2010).

Q Source: UNFCCC: CDM

74. Which of these taxes are accounted under the Direct tax
system?
1. Wealth tax
2. Service tax
3. Capital Gains Tax
4. Minimum Alternative Tax (MAT)

Select the correct answer using the codes below.

a) 1, 2 and 3 only
b) 1 only
c) 1, 3 and 4 only
d) 2 and 3 only


INSIGHTS PRELIMS TEST SERIES 2017

www.insightsonindia.com www.insightsias.com
TEST – 13 Solutions

Solution: c)
Justification: Statement 1: Is charged on the net wealth, which is sum
total of all taxable assets clubbed together, minus the amount of debt
owed.

Net wealth = (All assets) – (all debt). Wealth tax has been abolished
recently.

Statement 2: This in an indirect tax, for which the ultimate burden falls
on the consumers.

Statement 3: It is a direct tax on capital gains. If the capital gain is on


sale of property asset which is in the possession of seller for less than
three years duration then it is short term capital gain (STCG) else it falls
in the purview of long term capital gain (LTCG).

Statement 4: MAT is a kind of direct corporate tax. Earlier many


companies showed lower book profits and avoided taxes. But, after the
Mat provision was brought in, all companies having book profits under
the Companies Act shall have to pay a minimum alternate tax at 18.5%.
MAT is a way of making companies pay minimum amount of tax. It is
applicable to all companies except those engaged in infrastructure and
power sectors.

Q Source: Improvisation: Page 63: 12th Macroeconomics NCERT

75. The writ of Mandamus CANNOT be issued by the High Court


against
1. A private individual
2. Any government official
3. A subordinate court

Select the correct answer using the codes below.

a) 1 and 2 only
b) 1 and 3 only


INSIGHTS PRELIMS TEST SERIES 2017

www.insightsonindia.com www.insightsias.com
TEST – 13 Solutions

c) 1 only
d) 2 and 3 only

Solution: c)

Justification: It literally means ‘we command’. It is a command issued


by the court to a public official asking him to perform his official duties
that he has failed or refused to perform. It can also be issued against any
public body, a corporation, an inferior court, a tribunal or government
for the same purpose.

The writ of mandamus cannot be issued (a) against a private individual


or body; (b) to enforce departmental instruction that does not possess
statutory force; (c) when the duty is discretionary and not mandatory;
(d) to enforce a contractual obligation; (e) against the president of India
or the state governors; and (f) against the chief justice of a high court
acting in judicial capacity.

Q Source: Chapter 7: Indian Polity: M Laxmikanth

76. Assume you are the monetary authority of a nation. Your


nation’s currency has been steeply depreciating due to a variety of
reasons. How can you restore the currency value to its initial
position in the immediate term?
1. By tightening domestic monetary policy
2. By selling domestic currency in international market

Which of the above is/are correct?

a) 1 only
b) 2 only
c) Both 1 and 2
d) None

Solution: a)


INSIGHTS PRELIMS TEST SERIES 2017

www.insightsonindia.com www.insightsias.com
TEST – 13 Solutions

Justification: Statement 1: Tight monetary policy means interest rates


move upwards in the nation. A rise in the interest rates at home attracts
foreign investors who search for high returns on their bond investments.

As foreign capital inflow increases, it leads to a higher demand for


domestic currency. This is because foreign investors must buy domestic
currency and sell their foreign currency to invest in host nation. High
demand for domestic currency leads to its appreciation and the problem
is partly resolved. So, 1 is correct.

Statement 2: It should be the exact opposite. Selling domestic currency


will only bring its value lower in the international market as supply
exceeds demand for the currency. If, however, the Central bank starts
selling foreign currency (from its forex reserve) to buy domestic currency
on a large scale, this would increase demand for domestic currency and
help appreciate its value. So, 2 is incorrect.

Q Source: Page 81: 12th Macroeconomics NCERT

77.Section 141 and 144 of Criminal Procedure Code (1973) that are
often heard in news deal with
1. Reasonable restrictions on freedom of assembly
2. Powers of civilian administration to impose martial law

Which of the above is/are correct?

a) 1 only
b) 2 only
c) Both 1 and 2
d) None

Solution: a)
Justification: Under Sec 144, a magistrate can restrain an assembly,
meeting or procession if there is a risk of obstruction, annoyance or
danger to human life, health or safety or a disturbance of the public
tranquillity or a riot or any affray.


INSIGHTS PRELIMS TEST SERIES 2017

www.insightsonindia.com www.insightsias.com
TEST – 13 Solutions

Under Section 141 of the Indian Penal Code, as assembly of five or more
persons becomes unlawful if the object is (a) to resist the execution of
any law or legal process; (b) to forcibly occupy the property of some
person; (c) to commit any mischief or criminal trespass; (d) to force
some person to do an illegal act; and (e) to threaten the government or
its officials on exercising lawful powers.

Q Source: Current Affairs + lifted from Chapter 7: Indian Polity: M


Laxmikanth

78. Practice of agriculture can be said to have started in


a) Neolithic period
b) Chalcolithic period
c) Paleolithic Age
d) Mesolithic Age

Solution: a)
Learning: Earliest farming communities were characterized by
cultivation of plants & domestication of animals.

Option A: Earliest evident of Neolithic culture have been found at


Mehragarh on bank of river Bolan (Baluchistan) 7000 BC showing
beginning of agriculture & domestication of animals. So, A is correct.

Option D: Mesolithic Age is the transitional Phase between Paleolithic &


Neolithic Age.

People mostly lived on Hunting, Fishing & Food gathering; at later stage
also domesticated animals.

Chalcolithic period follows Neolithic and thus can’t be the answer.


Paleolithic Age lies much before, so C is also incorrect.

Q Source: Page 15: 11th Standard TamilNadu History Textbook


INSIGHTS PRELIMS TEST SERIES 2017

www.insightsonindia.com www.insightsias.com
TEST – 13 Solutions

79. Which of these constitutional provisions give the Judiciary


the power of Judicial Review?
1. Article 13 which says laws ultra vires the constitution shall be
void
2. Article 32 which gives Supreme Court the power to issue writs
3. Article 226 which gives High Courts the power to issue writs

Select the correct answer using the codes below.

a) 1 and 2 only
b) 2 and 3 only
c) 1 only
d) 1, 2 and 3

Solution: d)
Justification: Statement 1: Article 13 declares that all laws that are
inconsistent with or in derogation of any of the fundamental rights shall
be void. In other words, it expressively provides for the doctrine of
judicial review.

Statement 2 and 3: This power has been conferred on the Supreme Court
(Article 32) and the High Courts (Article 226) that can declare a law
unconstitutional and invalid on the ground of contravention of any of the
Fundamental Rights.

Q Source: Chapter 7: Indian Polity: M Laxmikanth

80. What is the importance of the Regulating Act of 1773 in the


Indian colonial administrative setup?
1. It was the first step taken by the British Government to control
and regulate the affairs of the East India Company in India.
2. It designated the Governor of Bengal as the ‘Governor-General
of India’ and vested in him major executive powers.
3. It provided for the establishment of a Supreme Court at Bombay
presidency.


INSIGHTS PRELIMS TEST SERIES 2017

www.insightsonindia.com www.insightsias.com
TEST – 13 Solutions

Select the correct answer using the codes below.

a) 1 and 2 only
b) 1 and 3 only
c) 2 and 3 only
d) 1 only

Solution: d)
Justification: Statement 2: It designated the Governor of Bengal as
the ‘Governor-General of Bengal’ and created an Executive Council of
four members to assist him. The first such Governor-General was Lord
Warren Hastings.

Statement 3: It was established at Calcutta (not Bombay) in 1774.

Learning: This act is of great constitutional importance as (a) it was


the first step taken by the British Government to control and regulate the
affairs of the East India Company in India; (b) it recognised, for the first
time, the political and administrative functions of the Company; and (c)
it laid the foundations of central administration in India.

• It made the governors of Bombay and Madras presidencies


subordinate to the governor-general of Bengal, unlike earlier, when
the three presidencies were independent of one another.

• It prohibited the servants of the Company from engaging in any


private trade or accepting presents or bribes from the ‘natives’.

• It strengthened the control of the British Government over the


Company by requiring the Court of Directors (governing body of
the Company) to report on its revenue, civil, and military affairs in
India.

Q Source: Chapter 1: Indian Polity: M Laxmikanth

81. Which of these constitutional bodies are functionally


autonomous from the government?
1. Election Commission of India


INSIGHTS PRELIMS TEST SERIES 2017

www.insightsonindia.com www.insightsias.com
TEST – 13 Solutions

2. Comptroller and Auditor-General of India


3. State Public Service Commissions

Select the correct answer using the codes below.

a) 1 only
b) 2 and 3 only
c) 1 and 3 only
d) 1, 2 and 3

Solution: d)
Justification: The Indian Constitution not only provides for the
legislative, executive and judicial organs of the government (Central and
state) but also establishes certain independent bodies. They are
envisaged by the Constitution as the bulwarks of the democratic system
of Government in India.

Statement 1: Election Commission ensures free and fair elections to the


Parliament, the state legislatures, the office of President of India and the
office of Vice-president of India.

Statement 2: Comptroller and Auditor-General of India audits the


accounts of the Central and state governments. He acts as the guardian
of public purse and comments on the legality and propriety of
government expenditure.

Statement 3: Union Public Service Commission to conduct examinations


for recruitment to all-India services and higher Central services and to
advise the President on disciplinary matters.

State Public Service Commissions in every state conduct examinations


for recruitment to state services and advice the governor on disciplinary
matters.

Q Source: Chapter 3: Indian Polity: M Laxmikanth


INSIGHTS PRELIMS TEST SERIES 2017

www.insightsonindia.com www.insightsias.com
TEST – 13 Solutions

82. It is generally said that Foreign Institutional Investors (FIIs)


have high “debt servicing burden”. What does it imply?
1. FII results in round tripping of laundered money.
2. FIIs bring hot money to primarily chase good returns in the
stock markets.

Which of the above is/are correct?

a) 1 only
b) 2 only
c) Both 1 and 2
d) None

Solution: b)
Justification: Statement 2: FIIs are also the most expensive in terms
of servicing burden (Kumar 2011). This is because they come to chase
primarily good returns at the stock markets and exchange rate
speculation. In 2007-08, Indian stock market was giving around 44%
return, which meant that for every dollar India received (as debt from
FIIs), it has to pay $1.44 in return resulting in 44% interest rate charge.
So, 2 is correct.

Statement 1: Whether it is laundered money or white money, both cause


a debt servicing burden. Even the money that does not come from tax
havens causes a debt burden. So, 1 is not an appropriate reason.

Q Source: Improvisation: Page 46: 12th Macroeconomics NCERT

83. The Menaka case (1978) is famously known for


a) Wider interpretation of the Article 21
b) Judicious rebalancing of fundamental rights and directive
principles
c) Evolution of the ‘rarest of rare’ doctrine
d) Clarifying the provisions related to President’s rule


INSIGHTS PRELIMS TEST SERIES 2017

www.insightsonindia.com www.insightsias.com
TEST – 13 Solutions

Solution: a)
Learning: In the Menaka case (1978), the Supreme Court overruled its
judgement in the Gopalan case by taking a wider interpretation of the
Article 21. Therefore, it ruled that the right to life and personal liberty of
a person can be deprived by a law provided the procedure prescribed by
that law is reasonable, fair and just.

• In other words, it has introduced the American expression ‘due


process of law’. In effect, the protection under Article 21 should be
available not only against arbitrary executive action but also
against arbitrary legislative action.

• Further, the court held that the ‘right to life’ as embodied in Article
21 is not merely confined to animal existence or survival but it
includes within its ambit the right to live with human dignity and
all those aspects of life which go to make a man’s life meaningful,
complete and worth living.

• It also ruled that the expression ‘Personal Liberty’ in Article 21 is of


the widest amplitude and it covers a variety of rights that go to
constitute the personal liberties of a man.

Q Source: Chapter 7: Indian Polity: M Laxmikanth

84. The Child Labour (Prohibition and Regulation) Amendment


Act, 2016 provides for
1. Prohibition of employment of children below 14 years in all
occupations or processes except where child helps his family.
2. Prohibition of employment of adolescents in hazardous
occupations
3. Resettlement and rehabilitation of children found in state of
bonded labour

Select the correct answer using the codes below.


INSIGHTS PRELIMS TEST SERIES 2017

www.insightsonindia.com www.insightsias.com
TEST – 13 Solutions

a) 1 only
b) 1 and 2 only
c) 2 and 3 only
d) 1, 2 and 3

Solution: b)
Justification: The Bill seeks to amend the Child Labour (Prohibition
and Regulation) Act, 1986 to widen the scope of the law against child
labour and stricter punishments for violations.

Statement 2: It provides for addition of a new category of persons called


“adolescent”. They are person between 14 and 18 years of age. Their
labour in hazardous industries is prohibited.

Government may confer powers on a District Magistrate (DM) to ensure


that the provisions of the law are properly carried out and implemented.

It also empowers the government to make periodic inspection of places


at which employment of children and adolescents are prohibited.

Statement 3: There is no R&R provision in the bill presently. This matter


is generally dealt by various schemes and policies, such as National Child
Policy and Bonded Labour rehabilitation scheme (covered in previous
tests).

Q Source: Recent acts and policies: Current Affairs

85. Consider the following statements about the administrative


situation in India before the enactment of the Charter Act 1833.
1. The Governor-General was appointed by the Court of Directors
of the East India Company (EIC).


INSIGHTS PRELIMS TEST SERIES 2017

www.insightsonindia.com www.insightsias.com
TEST – 13 Solutions

2. The Saint Helena Act temporarily abolished the offices of


Governor-General and Viceroy to promote decentralization in
India.

Which of the above is/are correct?

a) 1 only
b) 2 only
c) Both 1 and 2
d) None

Solution: a)
Justification: The Regulating Act of 1773 created the office with the
title of Governor-General of the Presidency of Fort William, or
Governor-General of Bengal to be appointed by the Court of Directors of
the East India Company (EIC).

• The Court of Directors assigned a Council of Four (based in India)


to assist The Governor General, and decision of council was
binding on Governor General during 1773-1784.

The Saint Helena Act 1833 (or Government of India Act 1833) re-
designated the office with the title of Governor-General of India. So, 2 is
incorrect.

• After the Indian Rebellion of 1857, the company rule was brought
to an end, and the British India along with princely states came
under the direct rule of the Crown. The Government of India Act
1858 created the office of Secretary of State for India in 1858 to
oversee the affairs of India, which was advised by a new Council of
India

Q Source: Chapter 1: Indian Polity: M Laxmikanth

86. If the total size of economy is growing year after year, it


implies that


INSIGHTS PRELIMS TEST SERIES 2017

www.insightsonindia.com www.insightsias.com
TEST – 13 Solutions

1. Growth rate of the economy must be increasing year on year.


2. Total investments must be increasing year on year.

Which of the above is/are correct?

a) 1 only
b) 2 only
c) Both 1 and 2
d) None

Solution: d)
Justification: GDP at market prices calculates total value of goods and
services produced within a year at market prices.

If it increases, it means entrepreneurs have decided to produce more


goods and services.

Statement 2: This can happen even without an increase in actual


investment, with the same machinery and labour. So, 2 isn’t necessarily
correct.

Statement 1: If the size of economy grows proportionately larger each


year, while the growth rate is positive, it may not necessarily be
increasing. For e.g. the nation may grow at 1% all the time and have a
larger GDP each year, but its growth rate has been constant. So, 1 is
incorrect.

Q Source: Page 28: Chapter 2: 12th Macroeconomics NCERT

87. High powered money consists of


1. Notes and coins in circulation with the public
2. Capitalization of all financial institutions
3. Deposits held by the Government of India with RBI

Select the correct answer using the codes below.

a) 1 and 2 only
b) 2 only


INSIGHTS PRELIMS TEST SERIES 2017

www.insightsonindia.com www.insightsias.com
TEST – 13 Solutions

c) 2 and 3 only
d) 1 and 3 only

Solution: d)
Justification: It is also called as monetary base.

It consists of currency (notes and coins in circulation with the public and
vault cash of commercial banks) and deposits held by the Government of
India and commercial banks with RBI.

If a member of the public produces a currency note to RBI the latter


must pay her value equal to the figure printed on the note.

Similarly, the deposits are also refundable by RBI on demand from


deposit-holders. These items are claims which the general public,
government or banks have on RBI and hence are considered to be the
liability of RBI.

Q Source: Page 43: 12th Macroeconomics NCERT

88. The Constitution confers which of the following rights and


privileges on the citizens of India but denies the same to aliens?
1. Right to education
2. Right to freedom of speech and expression
3. Right against exploitation

Select the correct answer using the codes below.

a) 2 and 3 only
b) 1 and 3 only
c) 2 only
d) 1, 2 and 3

Solution: c)

Justification: These rights are not available to the foreigners (friendly


aliens):


INSIGHTS PRELIMS TEST SERIES 2017

www.insightsonindia.com www.insightsias.com
TEST – 13 Solutions

• Right against discrimination on grounds of religion, race, caste, sex


or place of birth (Article 15).

• Right to equality of opportunity in the matter of public


employment (Article 16).

• Right to freedom of speech and expression, assembly, association,


movement, residence and profession (Article 19).

• Cultural and educational rights (Articles 29 and 30).

• Right to vote in elections to the Lok Sabha and state legislative


assembly.

• Right to contest for the membership of the Parliament and the


state legislature.

• Eligibility to hold certain public offices, that is, President of India,


Vice-President of India, judges of the Supreme Court and the high
courts, governor of states, attorney general of India and advocate
general of states.

Q Source: Chapter 7: Indian Polity: M Laxmikanth

89. Net National Product (NNP) is obtained after


a) Adding Net factor income from abroad to the Net Domestic
Product (NDP)
b) Discounting depreciation from Gross Domestic Product (GDP)
c) Adding net capital formation to Gross Domestic Product (GDP)
d) Summing the consumption expenditures of all individuals
residing within the economy

Solution: a)

Learning: GNP ≡ GDP + Factor income earned by the domestic factors


of production employed in the rest of the world – Factor income earned


INSIGHTS PRELIMS TEST SERIES 2017

www.insightsonindia.com www.insightsias.com
TEST – 13 Solutions

by the factors of production of the rest of the world employed in the


domestic economy

Hence, GNP ≡ GDP + Net factor income from abroad

If we discount (reduce) depreciation from GNP the measure of aggregate


income that we obtain is called Net National Product (NNP).

Q Source: Page 26: 12th Macroeconomics NCERT

90. Consider the following about Sustainable Development Goals


(SDGs) adopted from 2015.
1. It will apply to all UN member states and not just those in the
developing world.
2. SDGs consist of more goals but lesser number of targets as
compared to MDGs.

Which of the above is/are correct?

a) 1 only
b) 2 only
c) Both 1 and 2
d) None

Solution: a)
Justification: Statement 1: It will apply to all UN member states and
not just those in the developing world. In this way, we hope they will
become as “universal” as the Universal Declaration of Human Rights—a
vital element of the civic armoury in the fight for fairness.

Statement 2: SDGs consist of 17 goals and 169 targets in contrast to 8


goals and 18 targets in MDGs set to expire in 2015. So, 2 is incorrect.

Learning: Goals are related to ending poverty; sustainable


development; universal enrolment; improving implementation etc.


INSIGHTS PRELIMS TEST SERIES 2017

www.insightsonindia.com www.insightsias.com
TEST – 13 Solutions

These goals are similar to MDGs, while others like reducing inequality
are new to SDGs.

Instead of top-down approach in implementation of SDGs, bottom-up


approach is needed for the success of SDGs.

You can see all goals under the link in Q Source.

Q Source: http://www.un.org/sustainabledevelopment/sustainable-
development-goals/

91.Which of the following limit the effectiveness of Wholesale Price


Index (WPI) as an inflation targeting tool?
1. WPI excludes services sector.
2. WPI inflation does not necessarily reflect what a household
pays.

Which of the above is/are correct?

a) 1 only
b) 2 only
c) Both 1 and 2
d) None

Solution: c)
Justification: We have covered WPI and CPI broadly in previous tests,
hence this discussion is kept short and to the point.

Statement 1: WPI considers only goods and not services market which
count for nearly 65% of our economy. This seriously limits the
effectiveness of WPI.

Statement 2: Moreover, WPI rates are mostly captured from mandis or


places of wholesale business. Therefore, they don’t include prices at
household consumer level.


INSIGHTS PRELIMS TEST SERIES 2017

www.insightsonindia.com www.insightsias.com
TEST – 13 Solutions

WPI is also not consistent with global practices as most of the nations
either use CPI or PPI.

Q Source: Page 28-29: 12th Macroeconomics NCERT

http://www.agritech.tnau.ac.in/agriculture/agri_cropproduction_rice_s
ri.html

92. The Constitution of India declares it as “fundamental in the


governance of the country”. It is
a) Judicial review
b) Fundamental Rights
c) Universal Adult Franchise
d) Directive Principles of State Policy

Solution: d)
Learning: The directive principles are meant for promoting the ideal of
social and economic democracy. They seek to establish a ‘welfare state’ in
India.

However, unlike the Fundamental Rights, the directives are non-


justiciable in nature, that is, they are not enforceable by the courts for
their violation. Yet, the Constitution itself declares that ‘these principles
are fundamental in the governance of the country and it shall be the duty
of the state to apply these principles in making laws’.

Hence, they impose a moral obligation on the state authorities for their
application. But, the real force (sanction) behind them is political, that is,
public opinion.

In the Minerva Mills case (1980), the Supreme Court held that ‘the
Indian Constitution is founded on the bedrock of the balance between
the Fundamental Rights and the Directive Principles’.


INSIGHTS PRELIMS TEST SERIES 2017

www.insightsonindia.com www.insightsias.com
TEST – 13 Solutions

Q Source: Chapter 3: Indian Polity: M Laxmikanth

93. Consider the following about the Green Climate Fund (GCF).
1. GCF focuses on funding mitigation projects in developed
countries only.
2. The G-20 countries are not the party members of the GCF.
3. It supports countries in fulfilling their Intended Nationally
Determined Contributions (INDCs).

Select the correct answer using the codes below.

a) 1 and 2 only
b) 2 and 3 only
c) 3 only
d) 1 and 3 only

Solution: c)
Justification: GCF was established by 194 governments to limit or
reduce greenhouse gas emissions in developing countries. It, therefore,
must include G-20 members. So, 2 is incorrect.

The Green Climate Fund stands ready to support countries in


implementing their Nationally Determined Contributions, or NDCs –
through the multiple instruments that GCF can make available to
countries. So, 3 is correct.

The GCF supports projects mainly in developing countries as an


important financial mechanism of the UNFCCC. So, 1 is incorrect.

Q Source: UNFCCC: GCF


INSIGHTS PRELIMS TEST SERIES 2017

www.insightsonindia.com www.insightsias.com
TEST – 13 Solutions

94. Which of these are included in the “Current account”


transactions of India with the world?
1. Trade balance for goods
2. Flow of remittances
3. Trade in invisibles
4. Grants given by foreign governments

Select the correct answer using the code below.

a) 1, 2 and 4 only
b) 3 and 4 only
c) 1 and 2 only
d) 1, 2, 3 and 4

Solution: d)
Learning: The current account records exports and imports in goods
and services and transfer payments.

• Trade in services denoted as invisible trade (because they are not


seen to cross national borders) includes both factor income
(payment for inputs-investment income, that is, the interest,
profits and dividends on our assets abroad minus the income
foreigners earn on assets they own in India) and non-factor income
(shipping, banking, insurance, tourism, software services, etc.).

• Transfer payments are receipts which the residents of a country


receive ‘for free’, without having to make any present or future
payments in return.

• They consist of remittances, gifts and grants. They could be official


or private.

Q Source: Page 77: 12th Macroeconomics NCERT

95. Eighth Schedule recognizes which of the following


languages?


INSIGHTS PRELIMS TEST SERIES 2017

www.insightsonindia.com www.insightsias.com
TEST – 13 Solutions

1. English
2. Sanskrit
3. Kashmiri
4. Hindi

Select the correct answer using the codes below.

a) 3 and 4 only
b) 1 and 2 only
c) 2, 3 and 4 only
d) 1, 2, 3 and 4

Solution: c)
Justification: Originally, it had 14 languages but presently there are 22
languages.

They are: Assamese, Bengali, Bodo, Dogri (Dongri), Gujarati, Hindi,


Kannada, Kashmiri, Konkani, Mathili (Maithili), Malayalam, Manipuri,
Marathi, Nepali, Oriya, Punjabi, Sanskrit, Santhali, Sindhi, Tamil,
Telugu and Urdu.

Statement 1: English is not recognized under 8th schedule, even though it


is the official language of India. So, 1 is incorrect.

Sindhi was added by the 21st Amendment Act of 1967; Konkani,


Manipuri and Nepali were added by the 71st Amendment Act of 1992;
and Bodo, Dongri, Maithili and Santhali were added by the 92nd
Amendment Act of 2003.

Q Source: Chapter 3: Indian Polity: M Laxmikanth

96. As per RBI circulars, a bank can fulfil its Statutory Liquidity
Ratio (SLR) obligations in which of these forms?
1. Vault Cash
2. Gold holdings
3. Government security holdings

Select the correct answer using the codes below.


INSIGHTS PRELIMS TEST SERIES 2017

www.insightsonindia.com www.insightsias.com
TEST – 13 Solutions

a) 3 only
b) 1 and 2 only
c) 2 and 3 only
d) 1, 2 and 3

Solution: d)
Justification: The RBI Act instructs that all commercial banks (and
some other specified institutions) in the country have to keep a given
proportion of their demand and time deposits (NDTL or net demand and
time liabilities) as liquid assets in their own vault. This is called statutory
liquidity ratio.

• The word statutory here means that it is a legal requirement and


liquid asset means assets in the form of cash, gold and approved
securities (government securities).

• Theoretically, SLR is a monetary policy instrument (a direct


instrument). But at the practical level, SLR has helped the
government to sell its securities or debt instruments to banks.

Learning: Difference with CRR: Cash Reserve Ratio is the percentage


of the deposit (NDTL) that a bank has to keep with the RBI. CRR is kept
in the form of cash and that also with the RBI. No interest is paid on
such reserves.

On the other hand, SLR is the percentage of deposit that the banks have
to keep as liquid assets in their own vault.

Q Source: Page 41: 12th Macroeconomics NCERT

97. Which of the following are the benefits of using “Effective


Microorganisms” in the field?
1. They help in control of soil borne pathogens.
2. They help improve the photosynthetic potential of plants.

Which of the above is/are correct?


INSIGHTS PRELIMS TEST SERIES 2017

www.insightsonindia.com www.insightsias.com
TEST – 13 Solutions

a) 1 only
b) 2 only
c) Both 1 and 2
d) None

Solution: c)
Justification: Effective microorganisms (EM) are various blends of
common predominantly anaerobic microorganisms in a carbohydrate-
rich liquid carrier substrate.

Photosynthetic microorganisms, lactic acid bacteria, yeasts, plant growth


promoting Rhizobacteria and various fungi and Actinomycetes are some
of the examples.

In this consortium, each microorganism has its own beneficial role in


nutrient cycling, plant protection and soil health and fertility
enrichment.

Statement 1: For e.g. rhizobium may increase soil fertility, on the other
hand fungis may help control soil borne pathogens.

Statement 2: With synergistic and symbiotic relationships these EMs


may lead to an increase in the photosynthetic activity of plants.

Learning: Other benefits of EM use are

• Improved seed germination, seedling emergence, growth of plants,


flowering, fruiting and ripening of grains and fruits.

• Increased tolerance in plants against pest attack.

• Improves physico-chemical and biological properties of soil.

• Interdependent biological activity of different EM organisms


creates a congenial environment for growth and spread of soil's
flora and fauna. They also promote the growth and colonization of
VAM, which further help in plant growth promotion.

• Help in quick degradation of organic matter. With the use of EM


the requirement of compost can be reduced or dispensed with. Just
recycling of crop residue with EM can give similar results as with


INSIGHTS PRELIMS TEST SERIES 2017

www.insightsonindia.com www.insightsias.com
TEST – 13 Solutions

good compost. This saves lot of labour and space required for
compost preparation.

Q Source:
http://agritech.tnau.ac.in/org_farm/orgfarm_EM_tech_intro.html

98. Consider the following about ‘Climate Vulnerable Forum


(CVF).
1. It is a group of all island nations that are threatened with sea
level rise.
2. The group opposes international mechanisms on climate change
projects as it views them as non-participatory and exploitative.

Which of the above is/are correct?

a) 1 only
b) 2 only
c) Both 1 and 2
d) None

Solution: d)
Justification: The CVF is also known as Vulnerable Twenty’ (V20)
group.

Finance Ministers of the ‘Vulnerable Twenty’ (V20) representing close to


700 million people from 20 countries around the globe, hold their
inaugural meeting during the World Bank/International Monetary Fund
Annual Meetings last year. So, 1 is incorrect.

The new group called for a significant mobilization of public and private
finance for climate action at the international, regional and domestic
level ahead of the COP21 talks in Paris.

The V20 was created to share and scale up innovative approaches to


climate finance developed by those countries most affected by climate
change. So, 2 is clearly incorrect.


INSIGHTS PRELIMS TEST SERIES 2017

www.insightsonindia.com www.insightsias.com
TEST – 13 Solutions

Q Source:
http://www.worldbank.org/en/news/feature/2015/10/08/vulnerable-
twenty-ministers-more-action-investment-climate-resilience-low-
emissions-development

99. Elections to the Lok Sabha and the state assemblies will be
held on the basis of adult suffrage is a provision that was originally
instituted in
a) Fundamental Rights section of the Constitution
b) Representation of People Act, 1951
c) Part V of the Constitution that deals with the Parliament
d) None of the above

Solution: d)
Learning: There are two provisions in the Constitution that seek to
achieve political equality.

• One, no person is to be declared ineligible for inclusion in electoral


rolls on grounds of religion, race, caste or sex (Article 325).

• Two, elections to the Lok Sabha and the state assemblies to be on


the basis of adult suffrage (Article 326).

• Both these provisions are constitutiona, but cannot be found in


options A, B or C. So, all of them are wrong.

Therefore, the right to vote is basically a constitutional right, not a


fundamental right.

Q Source: Chapter 4: Indian Polity: M Laxmikanth


INSIGHTS PRELIMS TEST SERIES 2017

www.insightsonindia.com www.insightsias.com
TEST – 13 Solutions

100. A Mini clonal technology for plantation has been in practice


for years. It offers which of the following advantages as compared
to traditional nodal cutting technologies?
1. Uniform rooting ensures higher productivity in this technique.
2. Epigenetic variation is higher in this technique leading to
improved planting results.

Which of the above is/are correct?

a) 1 only
b) 2 only
c) Both 1 and 2
d) None

Solution: a)
Justification: Organizations such as TNPL have developed and
established a mini Clonal garden technology for mass multiplication of
elite clones at a faster rate compared to the traditional hedge garden
technology.

Statement 1: Under this method, rooting efficiency and the uniformity


has been increased significantly which resulted in uniformity in
establishment, growth and development of clones. For e.g. a mini clonal
garden can be maintained for longer periods.

Statement 2: It avoids epigenetic variation which proves superior to the


existing technology and gives better plantation results.

Epigenetics studies genetic effects not entirely encoded in the DNA


sequence of an organism. Such effects on cellular and physiological
phenotypic traits may result from external or environmental factors that
switch genes on and off and affect how cells express gene products.

If the variation in epigenetics is high, clonal plants may behave or


survive in an erratic fashion. By avoiding epigenetic variation, the mini
clonal technique gives better results.

Q Source:
http://agritech.tnau.ac.in/spl_tech_forestry/spl_technology_mini_clon
al_advantages.html


INSIGHTS PRELIMS TEST SERIES 2017

www.insightsonindia.com www.insightsias.com
TEST – 13 Solutions


INSIGHTS PRELIMS TEST SERIES 2017

Вам также может понравиться